Neurology and Stroke Flashcards

1
Q

An 84-year-old man has been brought into hospital because of self-neglect. He lives alone in a ground floor flat and has daily carer who have found him to be increasingly suspicious, accusing them of stealing and moving his property and becoming physically aggressive. In the past month he had been refusing to let them in. He also seemed to be experiencing auditory hallucinations and
had lost weight. Examination was normal except for BMI 19 and MMSE 18/30. Bloods, CXR, urine, cultures normal. CT is shown.
Which one of the following is most likely?
a. Alcoholic hallucinosis
b. Alzheimer’s disease
c. Delirium
d. Paranoid schizophrenia
e. Pick’s disease

A

b. Alzheimer’s disease

Alzheimer’s dementia can present as self-neglect and weight loss, especially when the patient is liv- ing alone. Paranoid ideation is also quite common and may be used by the patient as an explanation for symptoms of memory loss (e.g. misplacing items), as is physical aggression whereas auditory and visual hallucinations are less common. Alzhei- mer’s disease is the commonest cause of dementia, and most cases are sporadic; 5% of cases are inher- ited as an autosomal dominant trait mutations in the amyloid precursor protein (chromosome 21), presenilin 1 (chromosome 14), and presenilin 2 (chromosome 1) genes are thought to cause the inherited form. Risk of Alzheimer’s disease is increased in those with the apolipoprotein E allele E4 (present in 20% of population) is 15 times higher than those with two E3 alleles. Mild AD is characterized by minor behavioral changes, loss of memory of recent events (e.g. conversations, events), misplace items, struggle to find the right word in conversation, confused or lose track of day/date, difficulty planning and making decisions, visuospatial impairment, and lose interest in peo- ple or activities. Moderate AD will need reminders about self-care, increasingly forgetful, not recog- nize people, place themselves/others at risk (e.g. miss medication, leave gas stove on), easily upset/angry/aggressive, night-day reversal, agita- tion, socially inappropriate, delusions/hallucina- tions. Severe AD is characterized by increasing dependence on others for nursing care, bed/ wheelchair bound, weakness, unable to recognize familiar objects/people, incontinence, difficulty eating/swallowing and gradual loss of speech. Death is usually 8-10 years after symptom onset. Pathological changes include widespread cerebral atrophy, particularly involving the cortex and hip- pocampus. In AD, FDG-PET can show hypome- tabolism in the temporoparietal regions and/or the posterior cingulum. On microscopy there are cor- tical plaques due to deposition of type A-Beta- amyloid protein and intraneuronal neurofibrillary tangles caused by abnormal aggregation of the tau protein (excessive phosphorylation). There is also reduced acetylcholine due to damage to ascending forebrain projection, hence acetylcholinesterase inhibitors (donepezil, galantamine, and rivastig- mine) as options for managing mild to moderate Alzheimer’s disease. Memantine (a NMDA recep- tor antagonist) is reserved for patients with moder- ate—severe Alzheimer’s disease.

How well did you know this?
1
Not at all
2
3
4
5
Perfectly
2
Q

A 70-year-old gentleman attends outpatient clinic with his wife. She reports that her husband’s behavior has changed and that he has
become increasingly forgetful over the past year. He has gained 10 kg of weight over the past 6 months. His wife reports that he
has an uncontrollable appetite occasionally eating to the point of vomiting. She also states that he has a lack of interest when
the grandchildren visit. Over the last 4 weeks she has noticed that her husband has become more unsteady on his feet having had a number of falls. On examining him in clinic he has impaired word comprehension, reduced safety awareness on mobilizing and a positive palmomental reflex. There is no tremor, rigidity or shuffling gait. MMSE is 22/30. CT head is shown. Which one of the following is most likely?
a. Depression
b. Hypomania
c. Pick’s disease
d. Lewy body dementia
e. CJD

A

c. Pick’s disease

Frontotemporal lobar degeneration (FTLD) is the third most common type of cortical dementia after Alzheimer’s and Lewy body dementia. Common features of frontotemporal lobar dementias include: Onset <65 years, insidious onset, relatively preserved memory and visuospa- tial skills, personality change and social conduct problems. CT shows cortical loss in the frontal and temporal lobes, and FDG-PET/CT shows hypometabolism. There are three recognized types of FTLD:
1. Frontotemporal dementia (Pick’s disease). Most common type and is characterized by personality change and impaired social conduct. Other common features include hyperorality, disinhibition, increased appe- tite, and perseveration behaviors. Focal gyral atrophy (“knife-blade” atrophy) is characteristic of Pick’s disease and is
localized to frontal and temporal lobes only. Microscopic findings include Pick bodies, gliosis, neurofibrillary tangles, and senile plaques.
2. Progressive non-fluent aphasia (chronic progressive aphasia). Patients have non- fluent speech, they make short utterances that are agrammatic but comprehension is relatively preserved.
3. Semantic dementia: Here the patient has a progressive fluent aphasia but speech lacks content and conveys little meaning. Unlike in Alzheimer’s memory is better for recent rather than remote events.

How well did you know this?
1
Not at all
2
3
4
5
Perfectly
3
Q

A 64-year-old man presents with a 6 month history of abnormal behaviors which have been noticed by his wife. He has described
seeing vivid visual hallucinations of clowns in his living room which sometimes talk to him and appear very real. He believes that
he is the head of a circus and is about to go on a world tour although this is not true. At times he is lucid and is fully independent
but at other times he is disorientated in time and place and is unable to perform simple tasks such as preparing food and going to
the shops. His wife thinks that his mood is also lower since the onset of symptoms. He presented in A +E today because of having a
second fall in 2 weeks. There is no history of infective symptoms. He went to see his GP two days ago who thought that he may
have a UTI and prescribed trimethoprim. He has a history of stroke 10 years ago and hypertension and takes warfarin, amlodipine,
and enalapril. Physical examination is unremarkable except for slightly increased tone on the left side compared to the right. Which
one of the following is most likely?
a. Alzheimer’s disease
b. Semantic dementia
c. Hypothyroidism
d. Lewy body dementia
e. Schizophrenia

A

d. Lewy body dementia

Lewy body dementia is an increasingly recog- nized cause of dementia, accounting for up to 20% of cases. The characteristic pathological fea- ture is alpha-synuclein cytoplasmic inclusions (Lewy bodies) in the substantia nigra, paralimbic, and neocortical areas. The relationship between Parkinson’s disease and Lewy body dementia is complicated, particularly as dementia is often seen in Parkinson’s disease. Also, up to 40% of patients with Alzheimer’s have Lewy bodies. Neuroleptics should be avoided in Lewy body dementia as patients are extremely sensitive and may develop irreversible Parkinsonism. Features include progressive cognitive impairment, Par- kinsonism, and visual hallucinations (other fea- tures such as delusions and non-visual hallucinations may also be seen). Two out of three are needed for diagnosis. The visual hallu- cinations are often very vivid. He also has a few supportive features of Lewy body dementia hallu- cinations in other modalities, delusions, depres- sion and repeated falls. Diagnosis is usually clinical, but SPECT is increasingly used. It is cur- rently commercially known as a DaT scan. Dopa- minergic iodine-123-I FP-CIT is used as the radioisotope. The sensitivity of SPECT in diag- nosing Lewy body dementia is around 90% with a specificity of 100%. Currently, evidence best supports cholinesterase inhibitors in the treating of Lewy body dementia. It must be remembered that these patients have high sensitivity to neuro- leptics so Olanzapine should not be used here.

How well did you know this?
1
Not at all
2
3
4
5
Perfectly
4
Q

A 55-year-old man presents with cognitive decline over a 6-month period. He continues to progress and develops myoclonus and a left
hemiparesis. On examination, he is alert and orientated to time and place but appears easily startled every time you start a sentence. There is bilateral finger-nose and heel-shin dysmetria, mild postural tremor and mild speech slurring. Blood tests are normal including thyroid and liver function. Lumbar puncture: WCC<1, RBC 16, Protein 0.5 g/l, Glucose 3.4 mmol/l, gram stain negative, and no organisms cultured. An EEG demonstrated brief periodic spikes. A MRI head (FLAIR sequence) is shown. Which one of the following is most likely?
a. Alzheimer’s disease
b. Creutzfeldt-Jakob disease
c. Carbon monoxide poisoning
d. Huntington’s disease
e. Pick’s disease

A

b. Creutzfeldt-Jakob disease

Creutzfeldt-Jakob disease is a rapidly progressive spongiform encephalopathy due to accumulation
of prion proteins resistant to proteases. Sporadic CJD accounts for 85% of cases whereas 10-15% of cases are familial. The mean age of onset is 65 years, except for new variant CJD which affects younger patients (mean age 25 years). Fea- tures include dementia, myoclonic jerks (often stimulus-sensitive), startle response, and less commonly extrapyramidal signs. New variant CJD usually has psychological symptoms such as anxiety, withdrawal and dysphonia. MRI shows high signal on the cortical sulci surfaces (ribbon- ing) and increased signal in putamen and caudate head. EEG shows periodic spikes with sharp waves in sporadic CJD. CSF profile is usually normal but positive for 14-3-3 protein.

How well did you know this?
1
Not at all
2
3
4
5
Perfectly
5
Q

A 81-year-old male experiences progressive cognitive decline over the past 10 years. His wife reports that every 6 months or so
she will notice another significant decrease in his functioning. It is now at the point where he is aggressive and has little short-term memory. Past medical history includes hypertension and percutaneous coronary intervention after a myocardial infarction. Examination findings include poor attention and memory,
mild left hemiparesis (face, arm, and leg), and brisk reflexes throughout with extensor plantar reflex bilaterally and a shuffling
gait. Which one of the following is most appropriate?
a. Referral for subthalamic nucleus deep brain stimulation
**
c. Carotid endarterectomy
d. Commence carbidopa/levodopa
e. Commence memantine

A

b. Treat cardiovascular risk factors

Vascular dementia is one of the most common causes of dementia after Alzheimer’s disease, caus- ing around 15% of cases. The history and findings are most suggestive of a vascular dementia caused by multiple strokes, hence management of stroke risk factors is the primary option as there is no licensed treatment for it. Targeted treatment is made more difficult as there are multiple subtypes of vascular dementia depending on profile of ischemia: multi-infarct (cortical) dementia, small vessel (subcortical white matter) dementia, hypo- perfusion dementia (watershed infarcts), hemor- rhagic dementia, CADASIL, and mixed vascular- Alzheimer’s disease type.

How well did you know this?
1
Not at all
2
3
4
5
Perfectly
6
Q

A 55-year-old man presents with a 2-month
history of weakness in his right arm. He has
also noticed that his voice has become softer.
He is finding it hard to use door handles and
open jars. On two occasions his wife has
noticed him stumbling whilst walking. On
examination he has fasciculations over his
right deltoid muscle and wasting of the interossei muscles of the right hand. Power is 4/5 in
right shoulder abduction with absent reflexes
in the right arm but present elsewhere.
Coordination and sensation are normal with
a negative Romberg’s test. Which one of the
following is the most likely diagnosis?
a. Cervical myelopathy
b. Diabetic neuropathy
c. Amyotrophic lateral sclerosis
d. Multiple sclerosis
e. Hereditary sensory motor neuropathy

A

c. Amyotrophic lateral sclerosis

Motor neuron diseases (MND) result in progres- sive degeneration of upper motor neurons (Betz cells) and/or lower motor neurons (anterior horn cells). Genetic studies have implicated Cu/Zn superoxide dismutase-1 (SOD1) gene in sporadic cases. They rarely presents before 40 years of age. Various patterns/subtypes are recognized including: amyotrophic lateral sclerosis (Lou Geh- rig disease), progressive bulbar palsy (bulbar onset ALS), primary lateral sclerosis, spinal muscular atrophy, X-linked spinobulbar muscular atrophy (Kennedy disease) and hereditary spastic parapar- esis. ALS is the most common form and may present with limb symptoms (tripping, foot drop, wasting of the small hand muscles, wrist drop) in 75% or bulbar symptoms (slurred speech, hoarse- ness, decreased volume of speech, aspirating/chok- ing on meals) in 25% of cases. Progression of disease results in muscle atrophy, fasciculations, spasticity, muscle cramps, voice changes, dyspha- gia, dysarthria, and drooling. Other clues which
point towards a diagnosis of motor neuron disease include absence of sensory signs/symptoms, both UMN and LMN symptoms, no cerebellar signs, no ocular signs and abdominal reflexes are usually preserved and sphincter dysfunction is a late fea- ture. MRI is usually performed to exclude the dif- ferential diagnosis of cervical cord compression or cranial lesion, and shows T2 hyperintensity (better seen on FLAIR) along the length of the corticosp- inal tract. The diagnosis of motor neuron disease is clinical, but EMG and nerve conduction studies will show normal sensory conduction with abnor- mal spontaneous (fasciculation) and evoked muscle potentials. Riluzole is the only drug that has been proven to demonstrate a disease modifying effect in motor neurone disease, increasing survival from diagnosis from 12 to 15 months and should be started; therapies to reduce oxidative stress such as addition of vitamin E and N-acetylcysteine (NAC) are not recommended. Non-invasive ven- tilation is the only other therapy that seems to pro- long life expectancy but only if the patient can tolerate greater than 4 h of NIV per day and does not have severe bulbar dysfunction. NIV is recom- mended when the patient has developed signs of respiratory distress, type 2 respiratory failure, FVC<50% or the patient has reported orthop- nea/nocturnal hypoventilation. However, patients with severe bulbar palsy or cognitive impairment are excluded. Tracheostomy and long term inva- sive mechanical ventilation have been used in selected cases with respiratory deterioration despite being largely neurologically intact. Prog- nosis poor: 50% of patients die within 3 years

How well did you know this?
1
Not at all
2
3
4
5
Perfectly
7
Q

An 18-month-old girl presents with leg weakness. Tremors, primarily of the hands, had
been noted since 4 months of age. She was
crawling by 9 months of age and cruising
about the furniture by 12 months. Her language development was normal. Her 4-yearold sister was developing normally. Cranial
nerve examination was normal, and specifically, fasciculations of the tongue were not
noted. She was able to sit, crawl, and pull to a
stand. She could walk holding onto furniture
but could not walk independently. Deep tendon reflexeswere absent throughout, and there
were no Babinski signs. Sensory examination
was normal. Which one of the following is
the next appropriate test?
a. Serum ceruloplasmin
b. Electromyography
c. Nerve conduction studies
d. Survival motor neuron gene testing
e. MRI head

A

d. Survival motor neuron gene testing

Spinal muscular atrophy is a congenital lower motor neuron disorder manifesting as progressive, symmetric proximal muscular weakness occurring in 1 in 6000 to 1 in 10,000 births (second most common autosomal recessive disease in humans after cystic fibrosis). It is the leading inherited cause of infant death. Spinal muscular atrophy is classified clinically by the age at symptom onset and disease severity into type I (Werdnig- Hoffman disease, acute), type II (intermediate form, usually 7-18 months old and can sit unsup- ported but can’t walk independently), type III (Kugelberg-Welander disease, mildest form, pre- sents >18 months and able to achieve independent walking), and type IV SMA (adult-onset). Spinal muscular atrophy is inherited in autosomal reces- sive fashion or is sporadic. Mutations or deletions in the telomeric SMN (survival of motor neuron) gene occur in most patients. The loss of functional SMN protein results in premature neuronal cell death. The SMN protein has a role in cardiac development. If the history and physical examination suggest spinal muscular atrophy, a positive DNA test for deletion of the survival motor neuron gene eliminates the need for elec- trophysiological testing and muscle biopsy. How- ever, the SMN gene is deleted only in 96% of patients, serum creatine kinase activity may be 1 to 2 times normal. Electromyography reveals large motor units; nerve conduction velocities and sen- sory conduction times are normal, ruling out motor neuropathies. Muscle biopsy reveals group atrophy of type 1 and type 2 muscle fibers as opposed to the normal checkerboard pattern. In the most severe cases (Type I), children never gain the ability to sit unsupported and severe respira- tory problems mean children rarely survive beyond two years of age. Type II SMA may shorten life expectancy, but improvements in care standards mean the majority of people can live long, fulfilling, and productive lives. Survival into adulthood is now expected. Life expectancy is usu- ally unaffected in Types III and IV.

How well did you know this?
1
Not at all
2
3
4
5
Perfectly
8
Q

A 41-year-old man presents with confusion
and headaches for the last few weeks. He
was diagnosed with HIV 15 years ago and
has been stable on highly active antiretroviral
treatment. Other past medical history
includes an episode of Pneumocystis jirovecii
pneumonia 1 year ago. His latest CD4 count
is 29 cells/μl. An MRI (T1 C+) is shown. The
enhancing lesions on MRI show increased
uptake on Thallium-201 Chloride SPECT
scan. Which one of the following is likely to
be required?
a. Sulfadiazine + pyrimethamine
b. Dexamethasone
c. Methotrexate
d. Amphotericin B
e. Image guided aspiration and intravenous
antibiotics.

A

c. Methotrexate

Common focal cerebral lesions in HIV patients are toxoplasmosis (50%), primary CNS lymphoma (30%), and less commonly cerebral tuberculosis. Typically, PCNSL in immunocompetent individ- uals (whether HIV positive or not) will appear as a single homogenously enhancing lesion, or spread across the corpus callosum (butterfly pattern) and has a dramatic response to dexamethasone treatment hence is easier to differentiate from infection. In immunocompromised patients, how- ever, imaging appearances of PCNSL are more variable—with smaller lesions and faster growth outstripping blood supply leading to necrosis (ring-enhancing lesions) and making them chal- lenging to differentiate from other ring-enhancing lesions seen in immunocompromised individuals such as toxoplasma (multiple lesions) and tubercu- losis (usually only single abscess). Several limita- tions of diagnostic testing also complicate matters and are worthy of note. Firstly, even when primary CNS lymphoma presents with its classical, homogeneously enhancing imaging appearance a cytological diagnosis is still required before treat- ment with methotrexate can start. In this scenario, if there is little intracranial mass effect lumbar puncture can be performed and CSF cytology, flow cytometry, PCR for immunoglobin clonal gene rearrangements (to establish monoclonality) and EBV PCR (80% positive in AIDS-related PCNSL). Despite this, CSF is often non- diagnostic and serial samples may be required or alternatively brain biopsy which is the gold stan- dard. Additionally, many patients receive dexamethasone due to raised ICP or focal deficits which (i) further reduces the chance of diagnostic LP and (ii) may cause the lesion to “disappear” and prevent accurate brain biopsy. In those AIDS patients in whom diagnostic uncertainty between cerebral toxoplasma and PCNSL remains after non-diagnostic CSF, brain biopsy should be considered in the context of negative serological screening for toxoplasma and thallium-enhanced SPECT scan results (negative in toxoplasma, pos- itive in PCNSL). Treatment for toxoplasma includes sulfadiazine and pyrimethamine, whereas it is methotrexate chemotherapy for PCNSL (or radiotherapy as second-line therapy).

How well did you know this?
1
Not at all
2
3
4
5
Perfectly
9
Q

A 31-year-old man was diagnosed with HIV
5 years ago and had been taking highly active
antiretroviral therapy until 8 months ago when
he decided to stop. He had been doing well on
highly active antiretroviral therapy, but stopped
taking hismedications 8months ago because he
thought that he would be better off. Two
months ago, he was successfully treated for
Pneumocystis carinii pneumonia. He now presents with confusion and speech deficit. His
CD4 count is 155/ul. MRI appearances are
shown below. CSF PCR is positive for JC virus.
Which one of the following is most likely?
a. Adrenoleukodystrophy
b. Multiple sclerosis
c. Subacute sclerosing panencephalitis
d. Progressive multifocal leukoencephalopathy
e. AIDS dementia complex

A

d. Progressive multifocal leukoencephalopathy

Adrenoleukodystrophy, MS, SSPE, are all demye- linating diseases, but PML is the only one linked to JC virus in the context of immunocompromised patients (e.g. AIDS, post-transplantation). General- ized CNS disorders in patients with HIV include viral encephalitis, Cryptococcus meningitis, PML and AIDS dementia complex. Progressive multifocal leukoencephalopathy (PML) results in widespread demyelination due to infection of oligodendrocytes by JC virus (a polyoma DNA virus; papovavirus) resulting in subacute onset of behavioral changes, speech, motor, and visual impairment. On CT this may appear as single or multiple lesions, no mass effect, no enhancement but MRI clearly shows wide- spread high T2 and FLAIR signal. Encephalitis may be due to CMV or HIV itself rather than HSV. Cryptococcus is the most common fungal infection of CNS and presents with meningism, seizures, and focal neurological deficit with raised CSF pressure on LP and positive India Ink staining. AIDS demen- tia complex is caused by HIV itself (i.e. HIV enceph- alopathy/encephalitis) and correlates with high viral loads and the duration of the infection. With the use of HAART, a milder form of cognitive dysfunction, minor cognitive motor disorder (MCMD) has become common. MCMD accounts for approxi- mately 30% of patients with HIV infection, while HIV-associated dementia accounts for less than 10%. Imaging findings include widespread cortical atrophy, ventricular enlargement and white matter damage.

How well did you know this?
1
Not at all
2
3
4
5
Perfectly
10
Q

A 43-year-old man has been having nightly,
unilateral, throbbing headaches with the pain
focused at the back of his left eye. They have
been occurring daily for the past week. The
patient recalls having had a similar headache
5 years ago that lasted for several weeks. The
patient has noticed that the headache is associated with lacrimation and nasal congestion.
Which one of the following would be appropriate next in acute management?
a. Dihydroergotamine
b. Glyceryl trinitrate
c. Indometacin
d. Inhaled 100% oxygen
e. Propanolol

A

d. Inhaled 100% oxygen

This patient describes features of cluster head- ache, which is most likely to be terminated with inhalation of pure oxygen within minutes. Cluster headaches usually occur at night when the patient is asleep, and so practical access to the oxygen tank is possible. Propranolol is a β-adrenergic- blocking agent that is useful in the prophylaxis of some vascular headaches, but it is of no value in aborting a cluster headache. Dihydroergota- mine suppositories may abort some vascular headaches, but they do not have as obvious an effect in cluster as in classic or common migraine syndromes.

How well did you know this?
1
Not at all
2
3
4
5
Perfectly
11
Q

A 18-year-old female presents with a severe
right-sided throbbing headache associated
with nausea, vomiting, and photophobiawhich
failed to respond to ibuprofen. There are no
other neurological features in the history.
She has been having similar headaches 3-4
times per month for the past year. Her mother
had a similar problem. Her examination is
normal. Which one of the following would
be appropriate next in acute management?
a. Amitriptyline
b. Propanolol
c. Sumatriptan
d. Topiramate
e. Verapamil

A

c. Sumatriptan

This patient has common migraine (migraine without aura). Of the agents listed, only suma- triptan is generally considered of use to abort a headache. The triptans are a group of medica- tions that act as agonists at serotonergic recep- tors (specifically, the 5HT-1 receptors), and they have been found to be very effective at stop- ping migraine headaches. Additional agents that might be of benefit in abortive therapy include ibuprofen, aspirin, acetaminophen, isometheptene, or ergotamine. Several medica- tions are effective as prophylactic agents in the treatment of migraine. These include amitripty- line hydrochloride, propranolol, verapamil, and valproate. Verapamil and amitriptyline may be used as prophylactic (preventative) therapy. Most experts recommend initiating prophylactic therapy only when headaches occur at least one to two times per month. Metoclopramide hydro- chloride, sumatriptan, and ergotamine tartrate are appropriately used to treat an acute attack of migraine and should not be prescribed on a daily basis. Daily use of these medications can establish a rebound syndrome that results in a chronic daily headache.

How well did you know this?
1
Not at all
2
3
4
5
Perfectly
12
Q

A 45-year-old man is referred urgently to
hospital with a severe headache. The pain
had started gradually three days before and
was now severe. The patient reported the
headache was exacerbated by an upright posture with relief obtained by lying flat. Since
the headache started the patient had been
unable to stand for more than a few minutes
at a time but was reasonably comfortable
when lying down. The patient denied any
focal neurological symptoms and was constitutionally well. Clinical examination did not
demonstrate any focal neurological signs or
features of meningism. CT brain: no evidence of intra-axial or extra-axial bleeding;
no space occupying lesion; no hydrocephalus.
MRI brain with gadolinium: diffuse pachymeningeal enhancement without leptomeningeal
enhancement; subtle downward displacement
of brain on sagittal views. Which one of the
following would be appropriate next in acute
management?
a. Epidural blood patch
b. Flat bed rest
c. Laminectomy dural repair and sealant
d. Lumbar puncture
e. MRI whole spine with STIR

A

a. Epidural blood patch

The clinical presentation is consistent with spontaneous intracranial hypotension, in partic- ular given the strong relationship of pain to upright posture. The patient has known con- nective tissue disease and so is at increased risk for this diagnosis. The MRI brain with gadolin- ium images are characteristic for spontaneous intracranial hypotension and so confirm the diagnosis. Lumbar puncture is often difficult in spontaneous intracerebral hypotension and would not change initial management unless CSF infection needed excluding. Post lumbar
puncture headache (PLPH), a common compli- cation of a lumbar puncture (30%) and is thought to be caused by excess leakage of cere- brovascular fluid causing a relatively low intra- cranial pressure. Risk factors include: factors which may contribute to headache; increased needle size; direction of bevel; not replacing the stylet; increased number of LP attempts; factors that do not contribute to headache; increased volume of CSF removed; bed rest fol- lowing procedure; increased fluid intake post procedure; opening pressure of CSF; and posi- tion of patient. Management is conservative, including administering analgesia and sufficient fluids and caffeine. If this fails to resolve the headache after 72 h an epidural blood patch.

How well did you know this?
1
Not at all
2
3
4
5
Perfectly
13
Q

A 26-year-old female presents with difficulty
walking and complains of problems with her
vision in her right eye. She had an episode of
diarrhea a week ago, but has no other relevant
past medical history apart from problems with
her left eye 3 months earlier which had
resolved. On examination there is a right relative afferent papillary defect. Visual acuity and
color vision are 6/6 (20/20) with 17/17 Ishihara
plates on the left, and 6/60 (20/200) with 0/17
Ishihara plates on the right. She reports no diplopia with a full range of eye movements, no
facial weakness and normal facial sensation.
Fundoscopy was unremarkable. Examination
revealed 2/5 power on the left arm and leg in
all movements; and 4/5 in all movements in
right arm and leg, brisk reflexes bilaterally with
extensor plantar responses. There is patchy
loss of sensation to cotton wool on right
lateralwrist and anterior aspectleftlateral shin.
Anal tone and saddle sensation are intact. MRI
brain is normal and MRI spine (Sagittal T2
+T1 with gad) shown below. CSF shows
WCC 12/mm3
, RBC <1/mm3
, Glucose
4.5 mmol/dl, Protein 0.9 g/l, and negative for
oligoclonal bands.Which one of the following
tests is likely to be positive?
a. Anti-acetylcholine receptor antibody
b. Anti-aquaporin 4 antibody
c. Anti-muscle specific kinase antibody
d. Anti-voltage gated calcium channel antibody
e. Anti-voltage gated potassium channel
antibody

A

b. Anti-aquaporin 4 antibody

Neuromyelitis optica (NMO; Devic’s disease) is monophasic or relapsing-remitting demyelinating CNS disorder Although previously thought to be a variant of multiple sclerosis, it is now recognized to be a distinct disease, particularly prevalent in Asian populations. Features of optic neuritis include uni- lateral decrease in visual acuity over, poor discrim- ination of colors, “red desaturation,” pain worse on eye movement, relative afferent pupillary defect and a central scotoma. Diagnosis of NMO requires bilateral optic neuritis, transverse myelitis and 2 of the following:
1. Spinal cord lesion involving three or more spinal levels (longitudinally extensive trans- verse myelitis)
2. Initially normal MRI brain
3. Aquaporin 4 positive serum antibody (pos-
itive in 80%)
Adults are especially likely to develop a pattern more typical of relapsing-remitting MS after an initial episode of neuromyelitis optica. Management of acute episodes is with predniso- lone or plasma exchange, and recovery from acute optic neuritis usually takes 4-6 weeks. Long term treatment is with immunosuppression (e.g. azothiaprine, rituximab). Disease modifying drugs used in MS are not used in the treatment of NMO.

How well did you know this?
1
Not at all
2
3
4
5
Perfectly
14
Q

A 10-year-old girl presents with subacute
mental status change and left arm weakness.
She had a viral illness 1 week ago. On examination she appears drowsy. She has a left
sided hemiparesis with bilateral nystagmus.
Fundoscopy reveals papilledema. There are
no skin rashes. MRI head FLAIR sequence
is shown. MRI spine showed a longitudinally
extensive transverse myelitis. Which one of
the following is most likely?
a. Multiple sclerosis
b. Acute disseminating encephalomyelitis
c. Neurosarcoidosis
d. Neuromyelitis optica
e. Systemic lupus erythematosis
f. Lyme disease

A

b. Acute disseminating encephalomyelitis

Acute disseminated encephalomyelitis (ADEM, postinfectious encephalomyelitis) is a demyelin- ating disease of the brain, brainstem, and spinal cord that is indistinguishable from MS on MRI. It is, however, monophasic, meaning that it occurs acutely on a single occasion and not in a recurrent fashion like MS. It usually develops within days or weeks of a viral illness (e.g. scarlet fever, measles, chickenpox) or immunization. It is characterized by an acute onset of multifocal neu- rological symptoms with rapid deterioration, which can be fatal if untreated. Non-specific signs such as headache, fever, nausea, and vomiting may also accompany the onset of illness. Motor and sensory deficits are frequent and there may also be brainstem involvement including occulo- motor defects. The diagnosis is suggested by the MRI or CT picture of rapidly evolving white matter damage associated with a high ESR and a CSF under increased pressure with elevated red cell and white cell counts and elevated protein content. The CSF glucose content is usually normal. Management involves intravenous glu- cocorticoids and the consideration of IV immu- noglobulins where this fails.

How well did you know this?
1
Not at all
2
3
4
5
Perfectly
15
Q

A 35-year-old female presents with three days of increasing weakness in the right arm and reduced visual acuity in the left eye. She has had a similar episode 2 years ago which she recovered from completely. On examination
she has weakness in wrist extension and finger abduction in the left hand and visual acuity in the left eye was measured at 6/24 with an associated reduction in color saturation. Blood
tests were unremarkable. Her MRI scan is shown (Axial T1 with contrast and FLAIR). Which one of the following options should be used in acute management?
a. Commence high dose oral prednisone
and wean over a month
b. IV methylprednisolone
c. Natalizumab infusion
d. Interferon beta
e. Biopsy

A

b. IV methylprednisolone

Multiple sclerosis is a chronic, predominantly autoimmune demyelinating disease of the central nervous system (CNS) characterized by subacute neurologic deficit (relapses last at least 24 h) cor- relating with CNS lesions separated in time and space, excluding other possible disease. Peak pre- sentation at 20-40 years. Subtypes include: Relapsing-remitting MS (80%): relapses followed by complete or near-complete recovery, most of which later transition to secondary progressive MS during which there progression of disability with few or no relapses. Primary progressive MS (20%) shows progression of disability from the onset, rarely with relapses. Presentation is with optic neuritis, neurological symptoms related to transverse myelitis (e.g. bladder dys- function, myelopathy) or intracranial plaques of demyelination. Lhermitte’s sign is an electrical sensation radiating down the spine when the neck is passively flexed and is believed to signify spinal cord demyelination. Uhthoff’s phenomenon describes the worsening of MS symptoms with higher body temperature (e.g. hot weather, exer- cise, fever). Eye signs include nystagmus, RAPD
(Marcus Gunn pupil), and internuclear opthal- moplegia. Atypical presentations include trigem- inal neuralgia, seizures and acute psychiatric disturbance. Diagnosis of MS requires demon- stration of lesions disseminated in time and space (McDonald criteria), hence after a single episode it is termed “clinically indeterminate syndrome” (unless there is past medical history and old and newer lesions on MRI). If there are >3 white- matter lesions on MRI the 5-year risk of develop- ing multiple sclerosis is c. 50%. MRI features of demyelinating plaques include T1 hypointense (black holes), T2 and FLAIR hyperintense, and if new/active inflammation they enhance on T1 + GAD sequences. T2/FLAIR imaging may also show linear regions of perivenous demyelination perpendicular to the corpus callosum known as Dawson’s fingers. Other investigations include lumbar puncture which may show a raised protein and in 80% positive for oligoclonal bands (in CSF but not in serum) signifying increased intrathecal synthesis of IgG. Visual evoked potentials may be delayed, but well preserved waveform. Treatment in multiple sclerosis is focused at reducing dura- tion of relapses (acute) and reducing the fre- quency of relapses (disease modifying drugs) as there is no cure. For acute relapses high dose ste- roids (e.g. oral or IV methylprednisolone) may be given for 3-5 days to shorten the length of an acute relapse, although they do not alter the degree of recovery (i.e. whether a patient returns to baseline function).

How well did you know this?
1
Not at all
2
3
4
5
Perfectly
16
Q

A 43-year-old female presents with a second
episode of loss of sensation in her left anterior
thigh and right foot. This is her second episode
within the past 4 months. She had recently
reported an episode of left anterior shin numbness 1 year ago, when anMRI with gadolinium
8 NEUROLOGY AND STROKE 131
demonstrated “spots in her spinal cord” and
she was diagnosed with transverse myelitis.
Her past medical history also includes ulcerative colitis, diagnosed aged 27years old and primary sclerosing cholangitis. Routine bloods
are normal except for mild derangement of
liver function tests.Which one of the following
is most appropriate?
a. Interferon beta
b. Glatiramer acetate
c. Fingolimod
d. Natalizumab
e. Mitoxanthrone

A

b. Glatiramer acetate

Clinical and imaging features (focal lesion that does not exceed two vertebral segments in length and does not affect more than half the cross- sectional area of the cord) suggest relapsing- remitting MS with plaques in the cervical spinal cord. In general, current evidence suggests start- ing disease-modifying therapy at the point of diag- nosis of relapsing-remitting forms of MS since damage continues to occur (based on MRI studies) even between relapses. As such, disease modifying therapy is used in those with “active” relapsing MS defined either as 2 or more relapses in the last 2 years or one recent relapse and/or signs of new lesions on MRI. Interferon beta 1a or 1b can be used in relapsing remitting MS, secondary pro- gressive MS if there are still significant relapses and clinically isolated syndrome, but is contraindi- cated with deranged liver function. Glatiramer acetate may act as a myelin decoy for the immune
system and is not contraindicated in liver dysfunc- tion. Fingolimod is the only oral drug and is a sphingosine 1 phosphate receptor modulator affecting lymphocyte migration that has been proven to reduce number of relapses and slow the rate of number of new MRI lesions. However, it was also associated with increased incidence of varicella zoster, tumor formation, and progressive multifocal leucoencephalopathy (PML) hence reserved for patients who fail 1st line therapies. Similarly, while natalizumab is effective in modi- fying multiple sclerosis progression, it is also asso- ciated with PML and not considered a 1st line treatment. Mitoxanthrone is a chemotherapy agent that inhibits DNA synthesis and repair, associated with significant cardiotoxicity, reserved as last resort. Other problems may also need symp- tomatic treatment:
* Fatigue—exclude common causes (e.g. ane- mia, hypothyroid or depression), amantadine, mindfulness training and CBT.
* Spasticity—physiotherapy, baclofen, and gabapentin are first-line. Other options include diazepam, dantrolene, and tizanidine. Botox.
* Bladder dysfunction—ultrasound first to assess bladder emptying. If significant residual volume ! intermittent self-catheterization, whereas if no significant residual volume anti- cholinergics may improve urinary frequency.
* Oscillopsia may respond to gabapentin.

How well did you know this?
1
Not at all
2
3
4
5
Perfectly
17
Q

A 43-year-old female presents with a 2 week
history of mild left arm weakness and headache. MRI was done at presentation (shown).
She was discharged on dexamethasone 2 mg
twice daily due to her focal neurology with a
plan for awake craniotomy and resection. An
image guidance scan is repeated one week later
but there is no longer any ring-enhancement.
Which one of the following is most likely?
a. Cerebral abscess
b. High grade tumor
c. Metastasis
d. Primary CNS lymphoma
e. Demyelination

A

e. Demyelination

Tumefactive demyelination is inflammatory demyelinating disease which presents as a soli-
tary large (>2 cm) focus of demyelination within a cerebral hemisphere with associated edema that may simulate neoplasm or abscess. Presentation is acute ( 3 weeks) with headache, seizures, and focal neurologic deficits. Often it may be a monophasic episode of disease without recurrence, but some may evolve into relapsing- remitting MS. Imaging features in 50% have contrast enhancement in the form of an incom- plete ring, without enhancement at junctions with gray matter (or basal ganglia depending on orientation), and there is usually minimal mass effect. Advanced MR imaging techniques may be useful, and the rCBV values are significantly lower than for high-grade glial neoplasms. Man- agement is with high-dose corticosteroid therapy. Radiation or surgical excision of lesions misdiag- nosed as tumor will cause additional irreversible
neurologic deficits. Equally, ring-enhancing lesions in the setting of MS should not be consid- ered to be TDL—neoplasia and abscess should be excluded first.

How well did you know this?
1
Not at all
2
3
4
5
Perfectly
18
Q

A 31-year-old female suffered multiple cuts
and burns to both arms. On examination
there is marked wasting of brachioradialis
and the small muscles in both hands, with
reduced biceps and brachioradialis reflex.
She is weak in both arms, distally more than
proximally. Her lower limb and cranial nerve
examination is unremarkable. On testing
upper limb sensation, vibration and proprioception are intact but there appears to be
reduced pain and temperature sensation over
the C3/C4/C5 dermatomes. Which one of
the following is most likely?
a. Chiari malformation
b. Chronic inflammatory demyelinating
polyneuropathy
c. Guillain-Barré syndrome
d. Miller Fisher syndrome
e. Multiple sclerosis

A

a. Chiari malformation

The presence of loss of pain and temperature sen- sation in a “cape-like” distribution is highly sug- gestive of syringomyelia, which is commonly associated with Chiari I malformation. It may be slowly progressive, cause wasting and weak- ness of the arms, spinothalamic tract deficit (pain and temperature), loss of reflexes and upgoing plantars, and Horner’s syndrome.

How well did you know this?
1
Not at all
2
3
4
5
Perfectly
19
Q

A 64-year-old man presents with sudden
onset severe headache while watching television, followed by confusion and a tonicclonic seizure. Past medical history included
a 20 pack year smoking history, hypertension, hypercholesterolemia and myocardial
infarction two years ago requiring stenting.
On examination, GCS M5V4E3 but was protecting his own airway. Pupils were equal and
reactive. The patient was spontaneously
moving all his limbs and had downgoing
plantar reflexes. Cardiovascular, respiratory
and abdominal examination was unremarkable. Initial observations were blood pressure
220/115 mmHg, heart rate 89 beat/min, O2
sats (15 l O2) 100%, Respiratory rate 19/
min, temperature 37.1°C. CT brain is normal and lumbar puncture shows WCC 3/
mm3
, RBC 3, protein 0.6 g/l, glucose
5.4 mmol/l, and no xanthochromia. MRI is
shown (FLAIR). Which one of the following
is most likely?
a. Acute disseminated encephalomyelitis
b. Herpes simplex virus encephalitis
c. Multiple sclerosis
d. Posterior circulation stroke
e. Posterior reversible encephalopathy syndrome

A

e. Posterior reversible encephalopathy syndrome

Posterior reversible leucoencephalopathy syn- drome may present with thunderclap headache, usually followed rapidly by confusion, seizures and visual symptoms. The most common causes of PRES are hypertensive encephalopathy and eclampsia. Hypertension is commonly observed. CT brain and lumbar puncture results are usu- ally normal or near normal. The elevation of CSF protein with hypertensive encephalopathy is variable because intracranial hemorrhage may occur with the hypertensive crisis, but most patients will have moderate increases in CSF protein.
Diagnosis is made by evidence of vasogenic brain edema on MRI brain. PRES is often associ- ated with reversible cerebrovascular vasoconstric- tion syndrome with vasospasm on cerebral angiography. Management in this case will require blood pressure control in the critical care setting. Other causes of thunderclap headache include aneurysmal subarachnoid hemorrhage, cerebral venous sinus thrombosis, internal carotid artery dissection, pituitary apoplexy reversible cerebral vasoconstriction syndrome, and benign coital headache.

How well did you know this?
1
Not at all
2
3
4
5
Perfectly
20
Q

A 12-year-old boy with Lyme disease and
bilateral facial weakness is being treated with
a cephalosporin. The child’s facial strength
improves, but he notices twitching of the left
corner of his mouth whenever he blinks his
eye. This involuntary movement disorder is
probably an indication of which one of the
following?
a. Horner’s syndrome
b. Marcus Gunn phenomenon
c. Mononeuritis multiplex
d. Parinaud syndrome
e. Recurrent meningitis

A

b. Marcus Gunn phenomenon

Aberrant regeneration of a cranial nerve is not all that uncommon, but it is more often seen after injury to the third nerve than to the seventh. For unknown reasons, the regenerating motor fibers miss their original targets and innervate new destinations. With cranial ALS, facial twitch- ing occurs, but it is not preceded by unilateral weakness, and it is seen as the weakness evolves, not as it remits. Sarcoidosis may produce facial weakness with aberrant regeneration, but this patient’s history does not suggest this idiopathic granulomatous disease. There is nothing to sug- gest that his Lyme disease is recurring, although recurrent meningitis may develop with inade- quate treatment.

How well did you know this?
1
Not at all
2
3
4
5
Perfectly
21
Q

A 25-year-old woman has progressive gait
disorder. The initial physical examination
reveals hepatosplenomegaly and left sided
ataxia and abnormal finger-nose test. Urinalysis reveals proteinuria and microscopic
hematuria. Which one of the following findings is most likely?
a. Neurofibromas
b. Ash leaf spots
c. Retinal telangiectasia
d. Kayser-Fleisher rings
e. Facial angiofibromas

A

c. Retinal telangiectasia

The association of erythrocytosis with cerebellar signs, microscopic hematuria, and hepatospleno- megaly suggests von Hippel-Lindau syndrome. This hereditary disorder is characterized by polycystic liver disease, polycystic kidney disease, retinal angiomas (telangiectasia), and cerebellar tumors. This is an autosomal dominant inherited disorder with variable penetrance. Men are more commonly affected than women. Although neo- plastic cysts may develop in the cerebellum in persons with von Hippel-Lindau syndrome, these usually do not become sufficiently large to cause an obstructive hydrocephalus. Other abnor- malities that occur with this syndrome include adenomas in many organs. Hemangiomas may be evident in the bones, adrenals, and ovaries. Hemangioblastomas may develop in the spinal cord or brainstem, as well as in the cerebellum.

How well did you know this?
1
Not at all
2
3
4
5
Perfectly
22
Q

A 62-year-old female has discomfort in her
limbs and trouble getting off the toilet. She
is unable to climb stairs and has noticed a rash
on her face. On examination, she is found to
have weakness about the hip and shoulder
girdle. She has purplish-red discoloration of
the skin around her eyes, erythematous discoloration over the finger joints and purplish
nodules over the elbows and knees. Which
one of the following is the most likely
diagnosis?
a. Becker muscular dystrophy
b. Dermatomyositis
c. Inclusion body myositis
d. Myotonic dystrophy
e. Polymyositis

A

b. Dermatomyositis

Dermatomyositis is an inflammatory disorder causing a symmetrical, proximal muscle weakness and skin lesions. Polymyositis is a variant where skin lesions are not prominent. It may be idio- pathic, associated with connective tissue disorders or a paraneoplastic syndrome in about 20% of cases overall. Lung, ovarian, gastrointestinal tract, breasts, and other malignancies can cause it hence a thorough search for a primary is indi- cated. Skin manifestations include a lilac (helio- trope) rash around the eyes, photosensitive skin, macular rash over back and shoulder, Gottron’s papules, nail fold capillary dilatation and flat-topped purplish nodules over the elbows and knees. Other features are proximal muscle weakness with tenderness, Raynaud’s syndrome, respiratory muscle weakness, interstitial lung dis- ease and dysphagia/dysphonia.

How well did you know this?
1
Not at all
2
3
4
5
Perfectly
23
Q

A 67-year-old male is investigated for chest
pain and painful swallowing progressing over
the last few months with no response to proton pump inhibitors. There is no history of
weight loss or anorexia or smoking. On
examination you note a left-sided partial ptosis, and he reports diplopia on testing extrocular muscle movements. Sustained upward
gaze exacerbates his ptosis. There is no limb
muscle weakness or sensory disturbance.
CXR is shown. Which one of the following
tests is likely to be helpful?
a. Anti-acteylcholine receptor antibodies
b. Anti-GM1 antibody
c. Anti-GQ1b antibody
d. Anti-muscle specific kinase antibody
e. Anti-voltage gated calcium channel
antibody

A

a. Anti-acteylcholine receptor antibodies

Myasthenia gravis is an autoimmune damage that occurs at the neuromuscular junction, speci- fically at postsynaptic membrane acetylcholine
receptors. A functional acetylcholine deficiency develops at the synapse because receptors are blocked or inefficient. Myasthenia is more com- mon in women (2:1). Approximately 1/3 of patient have a thymoma (commonest tumor of anterior mediastinum) which can cause death by airway compression or cardiac tamponade. The chest X-ray shows a partially delineated mediastinal mass (anterior mediastinum) with regular borders, bulging the left upper mediastinal contour sugges- tive of a thymoma. Other associations include thymic hyperplasia and autoimmune disorders (pernicious anemia, autoimmune thyroid disor- ders, rheumatoid, SLE). Presentation is with ocular weakness (90%) including ptosis, opthalmoparesis generally worse with sustained upward gaze. More severe disease includes limb weakness, difficulty with swallowing, and respiratory difficulties. The key feature is muscle fatigability—muscles become progressively weaker during periods of activity and slowly improve after periods of rest. Patients usually report fatigue that increases as the day progresses. Investigations include single fiber electromyography (high sensitivity 92-100%), CT thorax to exclude thymoma, CK is normal, around 85-90% of patients have antibodies to acetylcholine receptors. In the remaining patients, about 40% are positive for anti-muscle-specific tyrosine kinase antibodies. Tensilon test (intrave- nous edrophonium) reduces muscle weakness temporarily—not commonly used anymore due to the risk of cardiac arrhythmia. Management includes initiating long-acting anticholinesterases (e.g. pyridostigmine), immunosuppression and thymectomy. Myasthenic crises (severe enough to require intubation) may be triggered by other med- ications, infection or other physiological stressors and necessitate plasma exchange or intravenous immunoglobulin.

How well did you know this?
1
Not at all
2
3
4
5
Perfectly
24
Q

A 65-year-old presents with a 3 month history of progressive weakness. She had initially noticed difficulty opening jars, but
now also has difficulty walking up stairs.
She denied any pain or sensory symptoms.
Past medical history included osteoporosis,
type 2 diabetes mellitus and hypertension.
8 NEUROLOGY AND STROKE 133
On neurological examination there were no
fasciculations, tone was normal and sensation
was intact. Power was reduced in finger flexion (3/5), wrist flexion (4/5), knee extension
(3/5), and hip flexion (4/5) bilaterally. Upper
limb reflexes were present but diminished,
but the knee jerk was absent and there were
flexor plantar responses bilaterally. There
was no tenderness over any muscle groups.
Cranial nerve examination was unremarkable. Blood results were normal except for
CRP 10 mg/l, ESR 41 mm/h, CK 290 u/l.
Which one of the following is most likely?
a. Diabetic amyotrophy
b. Inclusion body myositis
c. Polymyalgia rheumatica
d. Polymyositis
e. Chronic inflammatory demyelinating
polyneuropathy

A

b. Inclusion body myositis

Inclusion body myositis is the most common pri- mary myopathy in the elderly manifesting as a slowly progressive weakness, usually affecting fin- ger and wrist flexion initially (but does affect both proximal and distal muscles). Lower limb weakness may also occur with quadriceps. In upper and lower limbs flexors affected more than extensors. Reflexes are usually diminished as in other myopathies. Creatine kinase levels are usu- ally normal or only mildly raised, in contrast to polymyositis (where creatine kinase levels are usually markedly elevated). Associated with cyto- plasmic inclusions on muscle biopsy. Muscles are
often tender in polymyositis, the distal muscles are usually not affected until the disease is advanced and CK is significantly raised. Diabetic amyotrophy is characterized by painful wasting of the proximal lower limb muscles.

How well did you know this?
1
Not at all
2
3
4
5
Perfectly
25
Q

A 77-year-old male presents with a 2-day history of right temporal throbbing headache.
He has had migraines previously but never
this severe and usually occipital. There was
no other past medical history of note. On
examination, his right scalp is tender and a
prominent right temporal artery is noted.
He is apyrexic with no skin rashes. His blood
tests are as follows:Hb 13.1 g/dl, Plt 45010-
9/l, WCC 11.5, ESR 85, Na 142, K 4.0., Urea
10, Cr 118 umol/l, CRP 23 mg/l. Which one
of the following would you do next?
a. CT angiogram
b. Biopsy
c. Start prednisolone
d. Start azothiaprine
e. Carotid duplex ultrasound

A

c. Start prednisolone

Temporal (giant cell) arteritis. is large vessel vas- culitis which overlaps with polymyalgia rheuma- tica (PMR) typically in a patient >60 years old, usually rapid onset with evidence of head- ache and jaw claudication in many, and visual dis- turbances secondary to anterior ischemic optic neuropathy in the presence of a tender, palpable superficial temporal artery. In contrast, PMR usually presents with myalgia, morning stiffness in proximal limb muscles (not weakness), leth- argy, depression, low-grade fever, anorexia, night sweats with a raised ESR but normal CK. Inves- tigations for temporal arteritis include raised inflammatory markers: ESR >50 mm/hr and CRP may also be elevated, normal CK. Treat- ment with high dose prednisolone is started on clinical suspicion (due to the risk to vision) while awaiting temporal artery biopsy, and also because histology shows changes which characteristically “skips” certain sections of affected artery whilst damaging others hence a negative temporal artery biopsy does not rule out temporal arteritis. Patients with visual symptoms should be seen the same-day by an ophthalmologist as visual damage is often irreversible. If there is no response to prednisolone the diagnosis should be reconsidered.

How well did you know this?
1
Not at all
2
3
4
5
Perfectly
26
Q

A 65-year-old male has been diagnosed with small cell lung cancer and is currently undergoing chemotherapy. Over the last few months he
has noticed his vision deteriorating and complains of diplopia. He also feels weaker in his upper limbs although his symptoms do fluctuate. On examination he has mild ptosis of the eyelids bilaterally and a complex ophthalmoparesis affecting both eyes. He also has reduced power proximally in the upper limbs. Which one of the following may be associated
with this clinical picture?

a. Anti-Ro antibody
b. Anti-voltage gated potassium channel
antibody
c. Anti-voltage gated calcium channel
antibody
d. Anti-Hu antibody
e. Anti-GQ1b antibody

A

c. Anti-voltage gated calcium channel antibody

Lambert-Eaton myesthenic syndrome (LEMS) is a paraneoplastic myesthenic syndrome associ- ated with small-cell lung cancer where antibodies to voltage-gated calcium channels (VGCCs) have been reported in 75-100%. It presents similarly to myasthenia gravis, with proximal muscle weakness, hyporeflexia and autonomic features but does not show fatiguability of muscle strength—strength actually improves with greater effort.

How well did you know this?
1
Not at all
2
3
4
5
Perfectly
27
Q

A 17-year-old girl presents with a second episode on waking earlier in the morning where
she could not move at all for 2 h. She reports
no loss of consciousness and was aware
throughout the episode. There is no other significant past medical history or epilepsy. Routine systemic and neurological examination is
normal. A 12 lead ECG demonstrated a jerky
baseline with flat T waves.What one of the following is most likely?
a. Andersen-Tawil syndrome
b. Cataplexy
c. Hyperkalemia periodic paralysis
d. Hypokalemic period paralysis
e. Night terror

A

c. Hyperkalemia periodic paralysis

Periodic paralysis can be classified into hypoka- lemic and hyperkalemic periodic paralysis and Andersen-Tawil (long-QT) syndrome, due to mutations in skeletal muscle ion channels. Onset is most commonly in childhood and ado- lescents, with attacks of paralysis lasting hours and neurological examination is normally
unremarkable in between attacks. Diagnosis of hypokalemic periodic paralysis is often made clinically by episodes of paralysis, typically occur at night and may be triggered by carbo- hydrate meals, in association with low serum potassium but genetic testing can help if known mutations are present. Management is with lifelong potassium supplementation.

How well did you know this?
1
Not at all
2
3
4
5
Perfectly
28
Q

A 10-year-old presents to your neurology
clinic reporting 9 months of subtle and gradual
onset, progressive lower limb weakness. For
the past 18 months, he has noticed a difficulty
in keeping up with his peers in PE lessons,
which he initially put down to “not being very
sporty.” However, he feels weak whenever he
walks and has particular difficulty getting up
from a chair. His appearance is shown below.
Formal examination of power is 4/5 bilaterally
in shoulder abduction, adduction and normal
5/5 distally. 4/5 is also noted in hip flexion
and extension, 4+/5 in knee flexion and extension, 5/5 in ankle plantar and dorsiflexion. The
weaknesses demonstrated are not fatiguable
and are persistent. Reflexes are present in all
areas, plantars are downgoing. He has no other
past medical history. What is the likely
diagnosis?
a. Becker muscular dystrophy
b. Duchenne muscular dystrophy
c. Emery-Dreifuss syndrome
d. Facial-scapulo humeral syndrome
e. Limb-girdle dystrophy

A

b. Duchenne muscular dystrophy

Duchenne muscular dystrophy (DMD) and Becker muscular dystrophy (BMD) are X-linked recessive disorders caused by mutations in the dystrophin gene on Xp21 (DMD occurs in 1 in 3000-6000 live births; BMD is much less com- mon). Dystrophin is part of a large membrane associated protein in muscle which connects the muscle membrane to actin, part of the muscle cyto- skeleton. In DMD there is a frameshift mutation resulting in one or both of the binding sites are lost leading to a severe form while in BMD there is a non-frameshift insertion in the dystrophin gene resulting in both binding sites being preserved leading to a milder form. DMD usually presents with skeletal muscle weakness before the age of 5 years, calf pseudohypertrophy, Gower’s sign (child uses arms to stand up from a squatted posi- tion) and intellectual impairment (in 30%) which progresses if untreated such that boys become wheelchair-bound by their early teens. Histori- cally, death occurs by age 25 years, primarily from respiratory dysfunction and less often from heart failure. A multidisciplinary treatment approach including steroids, scoliosis surgery, ventilatory support, and cardiac therapy has improved sur- vival. BMD is associated with a more variable pre- sentation of skeletal muscle weakness from the age of 10 onwards and absence of intellectual impair- ment, and carries a better prognosis, with most patients surviving to the age of 40-50 years. Car- diac involvement is seen in both disorders, and the severity is not correlated with the severity of skeletal muscle involvement. CK is elevated to 5-10 times normal in both. Facioscapulohumeral muscular dystrophy is autosomal dominant and the third most common after the DMD and myo- tonic dystrophy. Muscle weakness tends to follow a slowly progressive but variable course. The patient initially presents with facial and/or shoulder girdle muscle weakness, which progresses to involve the pelvic musculature. The limb-girdle muscular dys- trophies are a group of disorders with a limb- shoulder and pelvic girdle distribution of weakness, but with otherwise heterogeneous inheritance and genetic cause. The onset of muscle weakness is var- iable but usually occurs before age 30 with com- plaints of difficulty with walking or running secondary to pelvic girdle involvement. As the disease progresses, involvement of the shoulder muscles and then more distal muscles occurs, with sparing of facial involvement. Emery-Dreifuss muscular dystrophy is a rare X-linked disorder in which skeletal muscle symptoms are often mild but with cardiac involvement that is both common and serious. It is characterized by a triad of early contractures of the elbow, Achilles tendon, and posterior cervical muscles; slowly progressing muscle weakness and atrophy, primarily in humer- operoneal muscles; and cardiac involvement.

How well did you know this?
1
Not at all
2
3
4
5
Perfectly
29
Q

A 55-year-old male known alcoholic is found confused on a street by a policeman not orientated in time or place. He is able to follow your commands in lifting his upper and lower limbs during his neurolog- ical exams. All reflexes were present. He fails to follow your finger with his eyes on cranial nerve examination and you note horizontal nystagmus. His gait is grossly ataxic.

Alcohol-related neurological disorders:
a. Alcoholic cerebellar degeneration
b. Alcoholic hallucinosis
c. Alcoholic neuropathy
d. Alcohol withdrawal seizures
e. Beriberi
f. Delerium tremens
g. Marchiafava-Bignami disease
h. Tobacco-alcohol amblyopia
i. Wernicke’s encephalopathy
j. Wernicke-Korsakoff syndrome

A

i. Wernicke’s encephalopathy

i, Wernicke encephalopathy. Thiamine deficiency can result in this classic triad of nys- tagmus, opthalmoplegia and ataxia requiring urgent replacement

How well did you know this?
1
Not at all
2
3
4
5
Perfectly
30
Q

A 45-year-old heavy drinker (30 units per day) presents 72 h after his last drink with agitation, and pointing around the room as if having hallucinations. He has a coarse tremor, sinus tachycardia at 120 bpm and sweating.
Alcohol-related neurological disorders:
a. Alcoholic cerebellar degeneration
b. Alcoholic hallucinosis
c. Alcoholic neuropathy
d. Alcohol withdrawal seizures
e. Beriberi
f. Delerium tremens
g. Marchiafava-Bignami disease
h. Tobacco-alcohol amblyopia
i. Wernicke’s encephalopathy
j. Wernicke-Korsakoff syndrome

A

f. Delerium tremens

How well did you know this?
1
Not at all
2
3
4
5
Perfectly
31
Q

A 60-year-old female is brought in by police as she was wandering and confused. She was disorientated in place and time, did not remember her birthday, was unable to recall three objects after 5 min and identified the hospital cleaner as her father and seemed to recognize people she had never met. On examination pupils were reactive, there was an ophthalmoparesis, nystagmus on attempted horizontal gaze and ataxic gait. Motor and sensory systems were normal.

Alcohol-related neurological disorders:
a. Alcoholic cerebellar degeneration
b. Alcoholic hallucinosis
c. Alcoholic neuropathy
d. Alcohol withdrawal seizures
e. Beriberi
f. Delerium tremens
g. Marchiafava-Bignami disease
h. Tobacco-alcohol amblyopia
i. Wernicke’s encephalopathy
j. Wernicke-Korsakoff syndrome

A

j. Wernicke-Korsakoff syndrome

j, Wernicke-Korsakoff syndrome. Repre- sents the addition of anterograde amnesia, ret- rograde amnesia and confabulation to Wernicke’s encephalopathy.

How well did you know this?
1
Not at all
2
3
4
5
Perfectly
32
Q

A 34-year-old male working in a feltprocessing plant develops tremors, memory
disturbances and personality change over
the course of months. On examination, he
has prominent gait ataxia, limb and facial
tremors, and decreased pain and temperature sense in his feet.

Poisoning:
a. Aluminum
b. Arsenic
c. Carbon monoxide
d. Cyanide
e. Ergot
f. Lead
g. Manganese
h. Mercury
i. Organophosphates
j. Thallium

A

h. Mercury

How well did you know this?
1
Not at all
2
3
4
5
Perfectly
33
Q

A 23-year-old volunteers abroad painting houses during a 3-month exchange. Towards the end of the trip he develops weakness in both wrists. On examination, there is bilateral wrist drop without any sen- sory deficit. An EMG reveals evidence of a peripheral motor neuropathy.

Poisoning:
a. Aluminum
b. Arsenic
c. Carbon monoxide
d. Cyanide
e. Ergot
f. Lead
g. Manganese
h. Mercury
i. Organophosphates
j. Thallium

A

f. Lead

How well did you know this?
1
Not at all
2
3
4
5
Perfectly
34
Q

A 45-year-old worker in an insecticide factory complains of severe stomach pain. She also has had problems with her memory, excessive drowsiness, and a senso- rimotor neuropathy with absent tendon reflexes.

Poisoning:
a. Aluminum
b. Arsenic
c. Carbon monoxide
d. Cyanide
e. Ergot
f. Lead
g. Manganese
h. Mercury
i. Organophosphates
j. Thallium

A

b. Arsenic

How well did you know this?
1
Not at all
2
3
4
5
Perfectly
35
Q

A 22-year-old farm worker has seizure. Neurological examination reveals fascicula- tions and occasional myoclonus. He is ataxic and has absent deep tendon reflexes. A sensory neuropathy is evident in his legs. Ulcers are evident on his fingers and toes. He says his diet was poor and mostly made food from his rye crop

Poisoning:
a. Aluminum
b. Arsenic
c. Carbon monoxide
d. Cyanide
e. Ergot
f. Lead
g. Manganese
h. Mercury
i. Organophosphates
j. Thallium

A

e. Ergot

How well did you know this?
1
Not at all
2
3
4
5
Perfectly
36
Q

A 38-year-old miner develops a shuffling gait, tremor, and drooling. His speech is difficult to understand and becomes quieter as he talks. On examination, cogwheel rigidity is evident in his arms and legs. His tremor is most evident when his limbs are at rest.

Poisoning:
a. Aluminum
b. Arsenic
c. Carbon monoxide
d. Cyanide
e. Ergot
f. Lead
g. Manganese
h. Mercury
i. Organophosphates
j. Thallium

A

g. Manganese

How well did you know this?
1
Not at all
2
3
4
5
Perfectly
37
Q

A 15-year-old boy has moderate mental retardation, attention deficit disorder, a long face, enlarged ears, and macroorchid- ism. Development has been steady but always at a delayed pace.

Genetic syndromes with learning disability:
a. Angelman’s syndrome
b. Cri du Chat syndrome
c. Down’s syndrome
d. Fragile X syndrome
e. Neurofibromatosis
f. Prader-Willi syndrome
g. Rett syndrome
h. Tuberous sclerosis
i. Velocardiofacial syndrome
j. Williams syndrome

A

d. Fragile X syndrome

How well did you know this?
1
Not at all
2
3
4
5
Perfectly
38
Q

An 11-year-old girl presents with obesity, excessive and indiscriminate gorging, small hands, feet, hypogonadism and mental retardation.

Genetic syndromes with learning disability:
a. Angelman’s syndrome
b. Cri du Chat syndrome
c. Down’s syndrome
d. Fragile X syndrome
e. Neurofibromatosis
**
g. Rett syndrome
h. Tuberous sclerosis
i. Velocardiofacial syndrome
j. Williams syndrome

A

f. Prader-Willi syndrome

39
Q

A 7-year-old boy is noted to have character- istic elfin facies, short stature and cardiovas- cular defects.

Genetic syndromes with learning disability:
a. Angelman’s syndrome
b. Cri du Chat syndrome
c. Down’s syndrome
d. Fragile X syndrome
e. Neurofibromatosis
f. Prader-Willi syndrome
g. Rett syndrome
h. Tuberous sclerosis
i. Velocardiofacial syndrome
j. Williams syndrome

A

j. Williams syndrome

40
Q

A 13-year-old boy has a history of thymus abnormalities, ear deformities, cleft palate, cardiac defects and short stature. He has annual blood tests to assess serum calcium.

Genetic syndromes with learning disability:
a. Angelman’s syndrome
b. Cri du Chat syndrome
c. Down’s syndrome
d. Fragile X syndrome
e. Neurofibromatosis
f. Prader-Willi syndrome
g. Rett syndrome
h. Tuberous sclerosis
**
j. Williams syndrome

A

i. Velocardiofacial syndrome

41
Q

A 5-year-old girl with progressive deterio- ration in cognitive function and loss of lan- guage displays stereotypic hand movements.

Genetic syndromes with learning disability:
a. Angelman’s syndrome
b. Cri du Chat syndrome
c. Down’s syndrome
d. Fragile X syndrome
e. Neurofibromatosis
f. Prader-Willi syndrome
g. Rett syndrome
h. Tuberous sclerosis
i. Velocardiofacial syndrome j. Williams syndrome

A

g. Rett syndrome

41
Q

A 48-year-old male develops weakness of his left wrist extensors and digits over one week, followed by involvement of his right hand and foot over the subsequent 6 weeks. On examination, he has wasting and 2/5 power in the left wrist and digit extensors. There is evidence of clawing of the right ring and little fingers along with wasting of the small muscles of the right hand (except the thenar eminence and the first two lumbricals). He has a right foot drop along with wasting of the anterior tibial and perineal muscles on that side. Fascicu- lations are seen in all of the areas of weak- ness. Sensory examination and reflexes are normal, no clonus and a flexor plantar response.

Neuropathy:
a. Acute intermittent porphyria
b. Charcot-Marie Tooth
c. Chronic axonal neuropathy
d. Chronic inflammatory demyelinating
polyneuropathy
e. Diabetic neuropathy
f. Hereditary sensory and autonomic neuropathy
g. HIV neuropathy
h. Mononeuritis multiplex
i. Multifocal motor neuropathy with conduction block
j. Paraneoplastic neuropathy
k. Paraproteinemic neuropathy
l. Sarcoidosis
m. Vasculitis

A

i. Multifocal motor neuropathy with conduction block

Multifocal motor neuropathy with con- duction block (MMNCB). Younger to middle-aged men who develop focal arm weakness in the distribution of a named nerve relatively rapidly (e.g. a week). Over several months additional named motor nerves become involved asymmetrically such that it may resemble MND (MMNCB shows conduction block due to segmental demye- lination. MND does not. MMNCB is a demyelinating condition, MND is axonal). Nerve conduction studies help to decide whether a motor neuropathy is axonal or demyelinating. Anti-GM1 antibodies may also be present. MMNCB, Guillain-Barré, and CIDP are all examples of demyelinating neuropathies, all of which therefore respond to intravenous immunoglobulin (IVIG).

41
Q

A 56-year-old female has a 6 month history of worsening numbness and paresthesias distally in the hands and feet, as well as proximal muscle weakness. Bulbar muscles are normal. An EMG shows multifocal con- duction block, slowing of nerve conduction, and minimal loss of amplitude of muscle action potentials. CSF examination shows an elevation in protein, but no increase in the number of cells.

Neuropathy:
a. Acute intermittent porphyria
b. Charcot-Marie Tooth
c. Chronic axonal neuropathy
d. Chronic inflammatory demyelinating
polyneuropathy
e. Diabetic neuropathy
f. Hereditary sensory and autonomic neuropathy
g. HIV neuropathy
h. Mononeuritis multiplex
i. Multifocal motor neuropathy with conduction block
j. Paraneoplastic neuropathy
k. Paraproteinemic neuropathy
l. Sarcoidosis
m. Vasculitis

A

d. Chronic inflammatory demyelinating

Similar to Guillain-Barré syndrome (acute inflamma- tory demyelinating polyneuropathy, AIDP), but it takes a slowly progressive or remitting course rather than one of acute onset. It is a polyradiculoneuropathy affecting the proxi- mal portions of the nerves where they exit the spinal cord (i.e. nerve root). Because nerve roots are affected, patients experience proximal and distal weakness and sensory loss from the onset. Raised CSF protein may be present due to the inflammatory response affecting nerve roots within the thecal sac.

42
Q

A 40-year-old male has a prophylactic dose of phenytoin for 7 days after conservative management of head injury. He presents with confusion, psychosis, abdominal pain, and vomiting. On examination, he is tachy- cardic, hypertensive, and febrile and appears delirious. His arms are weak and areflexic but sensation is relatively preserved.

Neuropathy:
a. Acute intermittent porphyria
b. Charcot-Marie Tooth
c. Chronic axonal neuropathy
d. Chronic inflammatory demyelinating
polyneuropathy
e. Diabetic neuropathy
f. Hereditary sensory and autonomic neuropathy
g. HIV neuropathy
h. Mononeuritis multiplex
i. Multifocal motor neuropathy with conduction block
j. Paraneoplastic neuropathy
k. Paraproteinemic neuropathy
l. Sarcoidosis
m. Vasculitis

A

a. Acute intermittent porphyria

Acute intermittent porphyria. A disor- der of heme biosynthesis in the liver resulting from increased production and excretion of porphobilinogen and θ-aminolevulinic acid.
It is characterized by recurrent attacks of abdominal pain, psychosis, and neuropathy (motor and autonomic). Autonomic neurop- athy results in gastroparesis, constipation/ pseudoobstruction (hence abdominal pain), and autonomic instability. Attacks may be provoked by drugs, such as barbiturates, anti- convulsants, sulfonamide antibiotics, and estrogens. Treatment is best accomplished with use of intravenous hematin when sup- portive measures are not adequate or the case is severe.

43
Q

A 25-year-old woman with a prior history of visual loss in the left eye and a spastic gait develops impaired pain and temperature per- ception in her feet. She was diagnosed with multiple sclerosis (MS) shortly after her visual loss. Her left fundus reveals optic atro- phy, and her facial movements are asymmet- ric. Chest X-ray reveals large hilar lymph nodes. Mammogram reveals no apparent car- cinoma. Serum ACE is positive.

Neuropathy:
a. Acute intermittent porphyria
b. Charcot-Marie Tooth
c. Chronic axonal neuropathy
d. Chronic inflammatory demyelinating
polyneuropathy
e. Diabetic neuropathy
f. Hereditary sensory and autonomic neuropathy
g. HIV neuropathy
h. Mononeuritis multiplex
i. Multifocal motor neuropathy with conduction block
j. Paraneoplastic neuropathy
k. Paraproteinemic neuropathy
l. Sarcoidosis
m. Vasculitis

A

l. Sarcoidosis

Neurosarcoid affects both central and peripheral nervous system, with optic atrophy, facial nerve palsies and peripheral neuropathy.

44
Q

A 55-year-old female presents with 3 weeks of bilateral tingling sensation in her medial one and half digits at night. She has noted a clawing of her 4th and 5th digits and she is particularly concerned by the cosmetic ele- ments. She also complains of a left sided foot drop present over the past 8 months. She has also had multiple admissions for surgery to her feet at childhood but she is unaware of further details. On exam- ination, she clinically has a left common peroneal palsy with bilateral thin calves, and loss of sensation in bilateral ulnar nerve territories.

Neuropathy:
a. Acute intermittent porphyria
b. Charcot-Marie Tooth
c. Chronic axonal neuropathy
d. Chronic inflammatory demyelinating
polyneuropathy
e. Diabetic neuropathy
f. Hereditary sensory and autonomic neuropathy
g. HIV neuropathy
h. Mononeuritis multiplex
i. Multifocal motor neuropathy with conduction block
j. Paraneoplastic neuropathy
k. Paraproteinemic neuropathy
l. Sarcoidosis
m. Vasculitis

A

f. Hereditary sensory and autonomic neuropathy

HSMN type I autosomal dominant due to defect in PMP-22 gene (which codes for myelin) result- ing in a predominantly demyelinating neu- ropathy. Features often start at puberty, motor symptoms predominate with distal muscle wasting, pes cavus, clawed toesfoot drop, and leg weakness. HSMN type II is pri- marily axonal neuropathy.

45
Q

A 29-year-old female with Type 1 diabetes mellitus presents with weakness in several muscles in different limbs. The pattern is lower motor neuron and does not fit with any particular peripheral, plexus, or root localization.

Neuropathy:
a. Acute intermittent porphyria
b. Charcot-Marie Tooth
c. Chronic axonal neuropathy
d. Chronic inflammatory demyelinating
polyneuropathy
e. Diabetic neuropathy
f. Hereditary sensory and autonomic neuropathy
g. HIV neuropathy
h. Mononeuritis multiplex
i. Multifocal motor neuropathy with conduction block
j. Paraneoplastic neuropathy
k. Paraproteinemic neuropathy
l. Sarcoidosis
m. Vasculitis

A

h. Mononeuritis multiplex

In this disorder, individual nerves are transiently disabled over the course of minutes to days, and the recovery of func- tion may require weeks to months. Diabetes is the commonest cause.

46
Q

A 67-year-old female has a 2 month history of progressive gait disturbance. On exami- nation, she has dysmetria of the limbs, a wide-based, unsteady gait; and hypermetric saccades. A hard, firm breast lump is discovered.

Paraneoplastic disorders:
a. Dorsal root ganglionopathy
b. Guillain-Barré syndrome
c. Hypercalcemia
d. Lambert-Eaton myasthenic syndrome
e. Limbic encephalitis
f. Motor neuron disease
g. Myasthenia gravis
h. Opsoclonus-myoclonus
i. Paraneoplastic cerebellar degeneration
j. Paraproteinemic neuropathy
k. Stiff man syndrome

A

i. Paraneoplastic cerebellar degeneration

Characterized by subacute, progressive ataxia, dysarthria, and nystagmus. Myoclo- nus, opsoclonus (irregular jerking of the eyes in all directions), diplopia, vertigo, and
hearing loss may also occur. The most com- mon associated tumor types are small cell carcinoma of the lung, ovarian/breast carci- noma, and lymphoma. Anti-Purkinje cell antibodies (anti-Yo antibodies) may be pre- sent in 50%. Paraneoplastic cerebellar degeneration may precede the symptoms of the underlying tumor itself

47
Q

A 70-year-old male with a history of lung cancer develops nausea and vomiting and then becomes lethargic. On examination, he is lethargic but arousable, disoriented, and inattentive. He is weak proximally and has diminished reflexes.

Paraneoplastic disorders:
a. Dorsal root ganglionopathy
b. Guillain-Barré syndrome
c. Hypercalcemia
d. Lambert-Eaton myasthenic syndrome
e. Limbic encephalitis
f. Motor neuron disease
g. Myasthenia gravis
h. Opsoclonus-myoclonus
i. Paraneoplastic cerebellar degeneration
j. Paraproteinemic neuropathy
k. Stiff man syndrome

A

c. Hypercalcemia

It may be a result of parathyroid- related peptide secreted by the tumor itself (usually lung cancer) or of bone destruction by metastatic disease. The elevated serum calcium decreases membrane excitability, leading to the clinical syndrome of fatigabil- ity, lethargy, generalized weakness, and are- flexia progressing to coma and even convulsions. Symptoms usually do not occur until levels reach 14 mg/dL (3 mmol/l) or higher.

48
Q

A 57-year-old female with a history of smoking has a 3-month history of hip and shoulder weakness. She also complains of xerostomia. There are no sensory symp- toms, and she is cognitively intact. On examination, she is orthostatic. There is proximal muscle weakness, but she has increasing muscle strength with repetitive activity of her muscles. Eye movements are normal.

Paraneoplastic disorders:
a. Dorsal root ganglionopathy
b. Guillain-Barré syndrome
c. Hypercalcemia
d. Lambert-Eaton myasthenic syndrome
e. Limbic encephalitis
f. Motor neuron disease
g. Myasthenia gravis
h. Opsoclonus-myoclonus
i. Paraneoplastic cerebellar degeneration
j. Paraproteinemic neuropathy
k. Stiff man syndrome

A

d. Lambert-Eaton myasthenic syndrome

Lambert-Eaton myasthenic syndrome (LEMS) shows subacute proximal muscle weakness and spares the bulbar mus- culature, and is due to presynaptic blockade
of voltage gated calcium channels by autoan- tibodies. A characteristic feature is the increase in strength briefly after repeated muscle activation. Most cases are associated with small cell lung cancer, or in the context of other autoimmune diseases.

48
Q

A 65-year-old female develops pain and paresthesias in her feet. On examination, she has stocking distribution sensory loss, and mild distal weakness with areflexia. Serum protein electrophoresis reveals a monoclonal gammopathy, and bone mar- row biopsy reveals plasma cell dyscrasia.

Paraneoplastic disorders:
a. Dorsal root ganglionopathy
b. Guillain-Barré syndrome
c. Hypercalcemia
d. Lambert-Eaton myasthenic syndrome
e. Limbic encephalitis
f. Motor neuron disease
g. Myasthenia gravis
h. Opsoclonus-myoclonus
i. Paraneoplastic cerebellar degeneration
j. Paraproteinemic neuropathy
k. Stiff man syndrome

A

j. Paraproteinemic neuropathy

Parapro- teinemic neuropathy. Polyneuropathy may occur in up to 15% of patients with multiple myeloma presents as a chronic distal sym- metrical sensory or sensorimotor neuropa- thy. CSF protein may be elevated if there is a chronic inflammatory demyelinating polyneuropathy-like picture. Up to 20% of patients referred for evaluation of polyneuro- pathy may a monoclonal gammopathy of unde- termined significance, but a hematologic malignancy may later declare itself.

49
Q

A 3-year-old boy presents with gait ataxia, choreoathetoid movements in his right hand and recurrent ear infections or epi- sodes of unexplained fever. On examination he had dilated venules on his ear, mild dys- arthria and a wide based ataxic gait. Deep tendon reflexes were absent.

Hereditary ataxia:
a. Abetalipoproteinemia
b. Ataxia telangiectasia
c. Ataxia with isolated vitamin E deficiency
d. Episodic ataxia type 1
e. Episodic ataxia type 2
f. Fragile X tremor ataxia syndrome
g. Friedreich’s ataxia
h. Machado-Joseph disease (SCA3)
i. Mitochondrial disorders
j. Spinocerebellar ataxias (SCA1

A

b. Ataxia telangiectasia

50
Q

A 24-year-old presents with progressive unsteadiness on walking over the past 6 months. Over the past 3 months, he has noticed a lack of articulation with his speech. On examination, his cardio- vascular, respiratory and abdominal sys- tems are normal. His finger-nose test is impaired bilaterally and he is unable to tandem walk. He denies any neck stiff- ness or headache. He has a full range of eye movements. He has absent reflexes in his lower limbs and upgoing plantars bilaterally.

Hereditary ataxia:
a. Abetalipoproteinemia
b. Ataxia telangiectasia
c. Ataxia with isolated vitamin E deficiency
d. Episodic ataxia type 1
e. Episodic ataxia type 2
f. Fragile X tremor ataxia syndrome
g. Friedreich’s ataxia
h. Machado-Joseph disease (SCA3)
i. Mitochondrial disorders
j. Spinocerebellar ataxias (SCA1

A

g. Friedreich’s ataxia

51
Q

A 36-year-old male presents with a 5-year history of increasing, progressive “clumsi- ness.” He cannot write legibly or even hold a key still using either hand to open a door. On examination, his cranial nerves were unremarkable except for mild multi- directional nystagmus at primary gaze. Fundoscopy was normal. Limb examina- tion revealed significant impairment of finger-nose and heel-shin testing. His gait, tone, power, sensation and reflexes were normal with downgoing plantars. A brief mini-mental state examination scored 30/30. Serum ANA was negative.

Hereditary ataxia:
a. Abetalipoproteinemia
b. Ataxia telangiectasia
c. Ataxia with isolated vitamin E deficiency
d. Episodic ataxia type 1
e. Episodic ataxia type 2
f. Fragile X tremor ataxia syndrome
g. Friedreich’s ataxia
h. Machado-Joseph disease (SCA3)
i. Mitochondrial disorders
j. Spinocerebellar ataxias (SCA1)

A

j. Spinocerebellar ataxias (SCA1)

52
Q

At age 5, a child is noted to have the loss of ankle jerks. At age 10, limb ataxia develops, followed by a peripheral neuropathy. Dur- ing adolescence, retinitis pigmentosa develops. Peripheral blood smear shows acanthocytosis.

Hereditary ataxia:
a. Abetalipoproteinemia
b. Ataxia telangiectasia
c. Ataxia with isolated vitamin E deficiency
d. Episodic ataxia type 1
e. Episodic ataxia type 2
f. Fragile X tremor ataxia syndrome
g. Friedreich’s ataxia
h. Machado-Joseph disease (SCA3)
i. Mitochondrial disorders
j. Spinocerebellar ataxias (SCA1)

A

a. Abetalipoproteinemia

53
Q

A 36-year-old man with tuberculosis is started on therapy with isoniazid, rifampin, and ethambutol. After 2 months his liver en- zymes are slightly deranged and he reports pins-and-needles sensations in his feet. Neurological examination reveals preserved power, but hypoactive deep tendon reflexes in the legs and impaired position sense.

Nutritional deficiency:
a. Folate
b. Niacin
c. Pyridoxine (vitamin B6) deficiency
d. Riboflavin
e. Thiamine (vitamin B1) deficiency
f. Vitamin A deficiency
g. Vitamin B12 deficiency
h. Vitamin C deficiency
i. Vitamin E deficiency
j. Vitamin D deficiency
k. Vitamin K

A

c. Pyridoxine (vitamin B6) deficiency

Patientsonantitu- berculous therapy with isoniazid may develop
isolated pyridoxine deficiency and subsequent peripheral neuropathy

54
Q

A 47-year-old known alcoholic female is found wandering and brought to the emer- gency room. She is disoriented to time, place, and person, but has no external evi- dence of head trauma. Examination reveals ataxic gait, paresis of conjugate gaze, and horizontal nystagmus. She does not have any ethanol in her bloodstream on testing.

Nutritional deficiency:
a. Folate
b. Niacin
c. Pyridoxine (vitamin B6) deficiency
d. Riboflavin
e. Thiamine (vitamin B1) deficiency
f. Vitamin A deficiency
g. Vitamin B12 deficiency
h. Vitamin C deficiency
i. Vitamin E deficiency
j. Vitamin D deficiency
k. Vitamin K

A

e. Thiamine (vitamin B1) deficiency

Symptoms suggest Wernicke’s encephalopathy

55
Q

A 59-year-old man developed progressive cramping of his legs, gait unsteadiness and paresthesia affecting his hands and feet over 1 year. He has also had some episodes of urinary incontinence. On examination, he has a spastic paraparesis with severe distur- bance of position and vibration sense in his legs, absent knee and ankle reflexes. Blood results show a megaloblastic anemia.

Nutritional deficiency:
a. Folate
b. Niacin
c. Pyridoxine (vitamin B6) deficiency
d. Riboflavin
e. Thiamine (vitamin B1) deficiency
f. Vitamin A deficiency
g. Vitamin B12 deficiency
h. Vitamin C deficiency
i. Vitamin E deficiency
j. Vitamin D deficiency
k. Vitamin K

A

g. Vitamin B12 deficiency

Symptoms suggest subacute combined degeneration of the cord.

56
Q

A 5-year-old boy develops progressive gait ataxia and limb weakness over the course of 3 months. Examination reveals diffusely absent deep tendon reflexes, proximal mus- cle weakness, ophthalmoparesis, and poor pain perception in the feet. Blood tests reveal elevated creatine kinase levels and evidence of liver disease without features of liver failure.

Nutritional deficiency:
a. Folate
b. Niacin
c. Pyridoxine (vitamin B6) deficiency
d. Riboflavin
e. Thiamine (vitamin B1) deficiency
f. Vitamin A deficiency
g. Vitamin B12 deficiency
h. Vitamin C deficiency
i. Vitamin E deficiency
j. Vitamin D deficiency
k. Vitamin K

A

i. Vitamin E deficiency

57
Q

Two brothers, 4 and 6 years of age, but not their 9-year-old sister, exhibit limb ataxia, nystagmus, and learning disability, and have abnormally low serum cortisol levels.

Leukodystrophies:
a. Acute disseminated encephalomyelitis
b. Alexander disease
c. CADASIL
d. Canavan disease
e. Cerebrotendinous xanthomatosis
f. Krabbe Disease
g. Metachromatic leukodystrophy
h. Pelizaeus-Merzbacher disease
i. Refsum disease
j. X-linked Adrenoleukodystrophy
k. Zellweger syndrome

A

j. X-linked Adrenoleukodystrophy

X-linked adrenoleukodystrophy. Pro- duces rapidly evolving brain damage in male infants or boys, with survival from onset of symptoms usually limited to 3 years. Long- chain fatty acids accumulate in adrenal corti- cal and other cells, resulting in adrenal insuf- ficiency and CNS disease

58
Q

A 3-month-old boy exhibits nystagmus and limb tremors unassociated with seizures. Over the next few years, he develops optic atrophy, choreoathetotic limb movements, seizures, and gait ataxia. He dies during sta- tus epilepticus and at autopsy is found to have widespread myelin breakdown with myelin preservation in islands about the blood vessels. The pathologist diagnoses a sudanophilic leukodystrophy to describe the pattern of staining observed on slides prepared to look for myelin breakdown products

Leukodystrophies:
a. Acute disseminated encephalomyelitis
b. Alexander disease
c. CADASIL
d. Canavan disease
e. Cerebrotendinous xanthomatosis
f. Krabbe Disease
g. Metachromatic leukodystrophy
h. Pelizaeus-Merzbacher disease
i. Refsum disease
j. X-linked Adrenoleukodystrophy
k. Zellweger syndrome

A

h. Pelizaeus-Merzbacher disease

Leukodystrophy with significant Sudan-staining typically become symptomatic during the first months of life, but survival may extend into the third decade of life. Most affected persons are male.

59
Q

A 17-month-old boy had developed nor- mally until approximately 13 months of age, when he began having progressive gait problems. On examination, the patient is spastic, yet nerve conduction studies (NCS) reveal slowed motor and sensory conduction velocities. Cerebrospinal fluid (CSF) protein is elevated. MRI reveals white matter abnormalities. Leukocyte test- ing reveals deficient arylsulfatase A activity.

Leukodystrophies:
a. Acute disseminated encephalomyelitis
b. Alexander disease
c. CADASIL
d. Canavan disease
e. Cerebrotendinous xanthomatosis
f. Krabbe Disease
g. Metachromatic leukodystrophy
h. Pelizaeus-Merzbacher disease
i. Refsum disease
j. X-linked Adrenoleukodystrophy
k. Zellweger syndrome

A

g. Metachromatic leukodystrophy

Sphingolipidosis due to arylsulfatase-A deficiency resulting in accumulation of galactosyl sulfa- tides. The affected person usually has retar- dation, ataxia, spasticity, and sensory disturbances usually symptomatic during infancy

60
Q

A 6-month-old child has a rapid regression of psychomotor function and loss of sight. There is increased urinary excretion of N- acetyl-L-aspartic acid.

Leukodystrophies:
a. Acute disseminated encephalomyelitis
b. Alexander disease
c. CADASIL
d. Canavan disease
e. Cerebrotendinous xanthomatosis
f. Krabbe Disease
g. Metachromatic leukodystrophy
h. Pelizaeus-Merzbacher disease
i. Refsum disease
j. X-linked Adrenoleukodystrophy
k. Zellweger syndrome

A

d. Canavan disease

Canavan disease may produce developmental regression at about 6 months of age, with extensor posturing, rigidity and myoclonic seizures may develop. There is accumulation of N-acetylaspartic acid in the blood and urine, but elevated levels in the brain establish the diagnosis.

61
Q

A 12-month-old develops progressive blindness and delayed cognitive milestones. Fundal examination shows a cherry red spot. There is a deficiency of hexaminosidase A.

Sphingolipidosis:
a. Batten’s disease
b. Cerebrotendinous xanthomatosis c. Fabry disease
d. Gaucher’s disease
e. GM1 gangliosidosis
f. Krabbe disease
g. Metachromatic leukodystrophy
h. Neimann-Pick disease
i. Sandhoff’s disease
j. Tay-Sachs disease
k. Wolman disease

A

j. Tay-Sachs disease

Thisisaganglioside storage disease that occurs more commonly in Ashkenazi Jews than in the general popu- lation. The early-onset form will produce macrocephaly and a cherry red spot in the fundus (retinal ganglion cells become dis- tended with glycolipid making retinal pale compared to fovea lacking ganglion cells). Children exhibit mental retardation, sei- zures, blindness and die prematurely

62
Q

An 8-month-old boy develops spasticity, head retraction, and difficulty swallowing. There is abnormal accumulation of gluco- cerebroside and the child will deteriorate and die within 3 years.

Sphingolipidosis:
a. Batten’s disease
b. Cerebrotendinous xanthomatosis
c. Fabry disease
d. Gaucher’s disease
e. GM1 gangliosidosis
f. Krabbe disease
g. Metachromatic leukodystrophy
h. Neimann-Pick disease
i. Sandhoff’s disease
j. Tay-Sachs disease
k. Wolman disease

A

d. Gaucher’s disease

63
Q

Non-paraneoplastic limbic encephalitis

Antibodies:
a. Anti-muscle specific kinase
b. Anti-NMDA receptor antibody
c. Anti-voltage gated calcium channel antibody
d. Anti-Aquaporin 4 antibody
e. Anti-Hu (ANNA-1)
f. Anti-Yo (PCA-1)
g. Anti-Ri (ANNA-2)
h. Anti-Tr
i. Anti-GAD
j. Anti-voltage gated potassium channel antibody
k. Anti-TA Ma2
l. Anti-GQ1b
m. Anti-myelin associated glycoprotein/sulfated glucoronul paragloboside
n. Anti-GD1b

A

j. Anti-voltage gated potassium channel antibody

64
Q

Neuromyelitis optica (Devic’s disease)

Antibodies:
a. Anti-muscle specific kinase
b. Anti-NMDA receptor antibody
c. Anti-voltage gated calcium channel
antibody
d. Anti-Aquaporin 4 antibody
e. Anti-Hu (ANNA-1)
f. Anti-Yo (PCA-1)
g. Anti-Ri (ANNA-2)
h. Anti-Tr
i. Anti-GAD
j. Anti-voltage gated potassium channel
antibody
k. Anti-TA Ma2
l. Anti-GQ1b
m. Anti-myelin associated glycoprotein/sul-
fated glucoronul paragloboside
n. Anti-GD1b

A

d. Anti-Aquaporin 4 antibody

65
Q

Guillain-Barré syndrome

Antibodies:
a. Anti-muscle specific kinase
b. Anti-NMDA receptor antibody
c. Anti-voltage gated calcium channel
antibody
d. Anti-Aquaporin 4 antibody
e. Anti-Hu (ANNA-1)
f. Anti-Yo (PCA-1)
g. Anti-Ri (ANNA-2)
h. Anti-Tr
i. Anti-GAD
j. Anti-voltage gated potassium channel
antibody
k. Anti-TA Ma2
l. Anti-GQ1b
m. Anti-myelin associated glycoprotein/sul-
fated glucoronul paragloboside
n. Anti-GD1b

A

l. Anti-GQ1b

66
Q

Myasthenia gravis

Antibodies:
a. Anti-muscle specific kinase
b. Anti-NMDA receptor antibody
c. Anti-voltage gated calcium channel
antibody
d. Anti-Aquaporin 4 antibody
e. Anti-Hu (ANNA-1)
f. Anti-Yo (PCA-1)
g. Anti-Ri (ANNA-2)
h. Anti-Tr
i. Anti-GAD
j. Anti-voltage gated potassium channel
antibody
k. Anti-TA Ma2
l. Anti-GQ1b
m. Anti-myelin associated glycoprotein/sul-
fated glucoronul paragloboside
n. Anti-GD1b

A

a. Anti-muscle specific kinase

67
Q

Lambert-Eaton myasthenic syndrome

Antibodies:
a. Anti-muscle specific kinase
b. Anti-NMDA receptor antibody
c. Anti-voltage gated calcium channel
antibody
d. Anti-Aquaporin 4 antibody
e. Anti-Hu (ANNA-1)
f. Anti-Yo (PCA-1)
g. Anti-Ri (ANNA-2)
h. Anti-Tr
i. Anti-GAD
j. Anti-voltage gated potassium channel
antibody
k. Anti-TA Ma2
l. Anti-GQ1b
m. Anti-myelin associated glycoprotein/sul-
fated glucoronul paragloboside
n. Anti-GD1b

A

c. Anti-voltage gated calcium channel
antibody

68
Q

A 4-year-old is found to have dwarfism, mental retardation, and clouding of his cor- neas. Tests show an α-L-iduronidase deficiency.

Mucopolysaccharidoses:
a. Hunter syndrome
b. Hurler syndrome
c. Sanfilippo syndrome A
d. Sanfilippo syndrome B
e. Sanfilippo syndrome C
f. Sanfilippo syndrome D
g. Maroteaux-Lamy syndrome h. Morquio syndrome A
i. Morquio syndrome B
j. Natowicz syndrome
k. Sly syndrome

A

b. Hurler syndrome

69
Q

A 6-year-old child is diagnosed with X- linked recessive mild learning difficulty due to iduronate sulfatase deficiency.

Mucopolysaccharidoses:
a. Hunter syndrome
b. Hurler syndrome
c. Sanfilippo syndrome A
d. Sanfilippo syndrome B
e. Sanfilippo syndrome C
f. Sanfilippo syndrome D
g. Maroteaux-Lamy syndrome h. Morquio syndrome A
i. Morquio syndrome B
j. Natowicz syndrome
k. Sly syndrome

A

a. Hunter syndrome

70
Q

A 7-year-old child presents with severe skele- tal dysplasia, short stature. Test shows abnor- mal accumulation of keratan sulfate secondary to deficiency of galactose-6-sulfate sulfatase.

Mucopolysaccharidoses:
a. Hunter syndrome
b. Hurler syndrome
c. Sanfilippo syndrome A
d. Sanfilippo syndrome B
e. Sanfilippo syndrome C
f. Sanfilippo syndrome D
g. Maroteaux-Lamy syndrome
h. Morquio syndrome A
i. Morquio syndrome B
j. Natowicz syndrome
k. Sly syndrome

A

h. Morquio syndrome A

Mucopolysaccharidoses are a group of metabolic disorders caused by the absence or malfunction- ing of lysosomal enzymes needed to break down glycosaminoglycans forming bone, cartilage,
tendons, corneas, skin, and connective tissue. Buildup of glycosaminoglycans collect in the cells, blood and connective tissues cause progres- sive cellular damage which affects appearance, tis- sue function and mental development. All are autosomal recessive except for Hunter syndrome. Treatment is with enzyme replacement.

71
Q

A 19-year-old male presents with diplopia, ataxia, and heart block.

Mitochondrial disorders:
a. Alpers-Huttenlocher syndrome
b. Ataxia neuropathy syndromes
c. Chronic progressive external ophthalmo-
plegia
d. Kearns-Sayre syndrome
e. Leber’s hereditary optic neuropathy
f. Leigh syndrome
g. MELAS
h. MEMSA
i. MERRF
j. NARP

A

d. Kearns-Sayre syndrome

72
Q

A 24-year-old male presents with reduced vision in both eyes over several weeks, and has developed “spasms” of his left hand and complains of palpitations. ECG is sug- gestive of ventricular pre-excitation.

Mitochondrial disorders:
a. Alpers-Huttenlocher syndrome
b. Ataxia neuropathy syndromes
c. Chronic progressive external ophthalmo-
plegia
d. Kearns-Sayre syndrome
e. Leber’s hereditary optic neuropathy
f. Leigh syndrome
g. MELAS
h. MEMSA
i. MERRF
j. NARP

A

e. Leber’s hereditary optic neuropathy

73
Q

A 34-year-old female with a history of migraines present’s with stroke-like epi- sodes affecting her left arm and leg, lactic acidosis, and cognitive impairment.

Mitochondrial disorders:
a. Alpers-Huttenlocher syndrome
b. Ataxia neuropathy syndromes
c. Chronic progressive external ophthalmo-
plegia
d. Kearns-Sayre syndrome
e. Leber’s hereditary optic neuropathy
f. Leigh syndrome
g. MELAS
h. MEMSA
i. MERRF
j. NARP

A

g. MELAS

Mitochondrial DNA is inherited maternally, and some of these disorders are thus transmitted from mother to children of both sexes. The mito- chondrial disorders, also termed mitochondrial
myopathies, encephalomyopathies, or respiratory chain disorders, are a heterogeneous group of dis- eases resulting from abnormalities in mitochon- drial DNA and respiratory chain function, hence manifest in tissue with a high respiratory workload such as brain, retina and skeletal mus- cle, especially extraocular and cardiac muscle

74
Q

A 73-year-old man steps out of the shower on a Saturday evening and is unable to remember that he and his wife have tickets to a play. He asks her repeatedly, “Where are we going”? He appears bewildered, but is alert, knows his own name, speaks fluently, and has no motor deficits. He has no history of memory disturbance and after 8 h returns to normal.

Dementia:
a. Alzheimer’s disease
b. B12 deficiency
c. Cortical Lewy body disease
d. Corticobasal degeneration
e. Creutzfeldt-Jacob disease
f. Depression
g. Frontotemporal dementia
h. HIV dementia complex
i. Huntington’s disease
j. Hypothyroidism k. Neurosyphilis
l. Normal pressure hydrocephalus
m. Transient global amnesia
n. Vascular dementia

A

m. Transient global amnesia

Episode of complete and reversible anterograde and ret- rograde memory loss lasting up to 24 h. Patients have a persistent loss of memory for the duration of the attack, which is char- acterized by bewilderment and repeating questions, but retained personal identity and ability to perform complex cognitive and motor tasks. TGA usually affects middle-aged or older men and often occurs in the setting of an emotional or other stressor, such as physical or sexual exertion. Although it shares features of transient ische- mic attack, it is not associated with an increased risk of stroke.

75
Q

A 50-year-old woman began having double vision and blurry vision 3 months ago and has since had diminishing interaction with her family, a paucity of thought and expres- sion, and unsteadiness of gait. Her whole body appears to jump in the presence of a loud noise. MRI is normal but CSF is pos- itive for 14-3-3 protein.

Dementia:
a. Alzheimer’s disease
b. B12 deficiency
c. Cortical Lewy body disease
d. Corticobasal degeneration
e. Creutzfeldt-Jacob disease
f. Depression
g. Frontotemporal dementia
h. HIV dementia complex
i. Huntington’s disease
j. Hypothyroidism k. Neurosyphilis
l. Normal pressure hydrocephalus m. Transient global amnesia
n. Vascular dementia

A

e. Creutzfeldt-Jacob disease

Creutzfeldt- Jakob disease. Patients may have ataxia, myoclonus, clumsiness, or dysarthria, as well as diplopia, distorted vision, blurred vision, field defects, changes in color perception, and visual agnosia. Ultimately, cortical blind- ness may occur. The diagnosis may be supported by the finding of periodic sharp waves at a 1-2 Hz frequency on EEG and the finding of elevated protein 14-3-3 in CSF

76
Q

A 17-year-old girl develops mild dementia, tremor, and rigidity. Her father died in his fourth decade of life of a progressive dementing illness associated with jerking (choreiform) limb movements. On expo- sure to L-dopa, she becomes acutely agi- tated and has jerking limb movements.

Dementia:
a. Alzheimer’s disease
b. B12 deficiency
c. Cortical Lewy body disease
d. Corticobasal degeneration
e. Creutzfeldt-Jacob disease
f. Depression
g. Frontotemporal dementia
h. HIV dementia complex
i. Huntington’s disease
j. Hypothyroidism k. Neurosyphilis
l. Normal pressure hydrocephalus m. Transient global amnesia
n. Vascular dementia

A

i. Huntington’s disease

77
Q

A 62-year-old man has had 2 years of pro- gressive memory loss and inappropriate behavior. He has been delusional. More recently, he has developed tremors, myoc- lonus, dysarthria, and unsteadiness of gait. The CSF shows a lymphocytic pleocytosis, protein of 150, and positive VDRL.

Dementia:
a. Alzheimer’s disease
b. B12 deficiency
c. Cortical Lewy body disease
d. Corticobasal degeneration
e. Creutzfeldt-Jacob disease
f. Depression
g. Frontotemporal dementia
h. HIV dementia complex
i. Huntington’s disease
j. Hypothyroidism
k. Neurosyphilis
l. Normal pressure hydrocephalus m. Transient global amnesia
n. Vascular dementia

A

k. Neurosyphilis

Neurosyphilis. General paresis is a chronic, often insidious meningoencephalitis that may be delayed up to 20 years after the original spi- rochetal infection. Clinically, it manifests as dementia, delusions, dysarthria, tremor, myoclonus, seizures, spasticity, and Argyll Robertson pupils. Other presentations of neurosyphilis include meningitis, meningo- vascular syphilis causing infarcts, optic atro- phy, and tabes dorsalis (characterized by ataxia, urinary incontinence, and lightning pains caused by degeneration of the posterior spinal roots)

78
Q

A 44-year-old woman presents with inat- tentiveness, poor concentration, weight gain and lethargy. She has paranoid delu- sions. There is mild proximal weakness and ataxia. On general examination, she has edema, coarse and pale skin, macroglos- sia and delayed relaxation of the ankle reflexes.

Dementia:
a. Alzheimer’s disease
b. B12 deficiency
c. Cortical Lewy body disease
d. Corticobasal degeneration
e. Creutzfeldt-Jacob disease
f. Depression
g. Frontotemporal dementia
h. HIV dementia complex
i. Huntington’s disease
j. Hypothyroidism
k. Neurosyphilis
l. Normal pressure hydrocephalus m. Transient global amnesia
n. Vascular dementia

A

j. Hypothyroidism

Hypothyroidism in adults may present with headache, dementia, psy- chosis, and decreased consciousness. Neuro- muscular findings are also common, and they include a proximal myopathic weakness and a delay in the relaxation phase of reflexes.

79
Q

A 36-year-old female complianed of a syn- copal episode. She described dizziness and fatigue on her way to her bedroom, shortly after she passed out. She regained con- sciousness after a few minutes, feeling tired and soaked in her sweat. Her PE was normal. Her resting ECG was within
normal. Routine laboratory work up was
normal. Echocardiography was normal.

Syncope:
a. Autonomic failure
b. Cardiac arrhythmia
c. Carotid sinus syncope
d. Cerebrovascular steal
e. Dehydration
f. Drug-induced orthostatic syncope g. Epileptic seizure
h. Hypoglycemia
i. Neurally mediated (vasovagal) syncope
j. Non-epileptic attack disorder
k. Situational syncope
l. Structural cardiac disease

A

i. Neurally mediated (vasovagal) syncope

80
Q

A 51-year-old female presents to the ER complaining of a sudden syncopal episode while climbing up the stairs. Her PE showed that she is in respiratory distress, BMI of 40, hemodynamically stable, cardiac examination showed normal. Routine laboratory work up was normal. Echocardi- ography was normal apart from mild MR
and mild TR.

Syncope:
a. Autonomic failure
b. Cardiac arrhythmia
c. Carotid sinus syncope
d. Cerebrovascular steal
e. Dehydration
f. Drug-induced orthostatic syncope g. Epileptic seizure
h. Hypoglycemia
i. Neurally mediated (vasovagal) syncope
j. Non-epileptic attack disorder
k. Situational syncope
l. Structural cardiac disease

A

b. Cardiac arrhythmia

81
Q

A 72-year-old with a 10-year history of
Parkinson’s disease presents with recurrent syncopal episodes on standing up from a sitting position.

Syncope:
a. Autonomic failure
b. Cardiac arrhythmia
c. Carotid sinus syncope
d. Cerebrovascular steal
e. Dehydration
f. Drug-induced orthostatic syncope g. Epileptic seizure
h. Hypoglycemia
i. Neurally mediated (vasovagal) syncope
j. Non-epileptic attack disorder
k. Situational syncope
l. Structural cardiac disease

A

a. Autonomic failure

82
Q

A 22-year-old complains of regular, right- sided throbbing headaches. Changes in her vision that precede the headache by 20 min include scintillating lights just to the left of her center of vision progressing to a blind spot which then clears before the headache starts. It rarely lasts more than 1 h, but is usu- ally accompanied by nausea and vomiting.

Headache:
a. Analgesic overuse headache b. Atypical facial pain
c. Cluster headache
d. Hypnic headache
e. Migraine with aura
f. Migraine without aura
g. Opthalmoplegic migraine h. Paroxysmal hemicranias
i. Postherpetic neuralgia
j. Retinal migraine k. Sinusitis
l. Short-lasting unilateral neuralgiform headache with conjunctival injection and tearing (SUNCT)
m. Tension headache
n. Trigeminal neuralgia
o. Vertebrobasilar migraine

A

e. Migraine with aura

Migraine with aura. Classic migraine is usually familial, involves a unilateral, throb- bing head pain, and diminishes in frequency with age. The blind spot, or scotoma, that may develop as part of the aura is usually hom- onymous hemianopia. It typically enlarges and may intrude on the central vision. The margin of the blind spot is often scintillating or daz- zling. Homonymous hemianoptic defects of the sort that develop during the aura of a classic migraine indicate an irritative lesion that is affecting one part of the occipital cortex in one hemisphere of the brain. Other focal neu- rological phenomena may precede classic migraine; the most common are tingling of the face or hand, mild confusion, transient hemiparesis, and ataxia. Fatigue, irritability, and easy distractibility often develop before a migraine. Affected persons usually also have hypersensitivity to light and noise during an attack

83
Q

A 35-year-old man has severe throbbing pain waking him from sleep at night and persisting into the day. This pain is usually centered about his left eye and appears on a nearly daily basis for several weeks or months each year. He becomes combative and agitated during the onset, but never vomits or develops focal weakness.

Headache:
a. Analgesic overuse headache
b. Atypical facial pain
c. Cluster headache
d. Hypnic headache
e. Migraine with aura
f. Migraine without aura
g. Opthalmoplegic migraine
h. Paroxysmal hemicranias
i. Postherpetic neuralgia
j. Retinal migraine k. Sinusitis
l. Short-lasting unilateral neuralgiform headache with conjunctival injection and tearing (SUNCT)
m. Tension headache
n. Trigeminal neuralgia
o. Vertebrobasilar migraine

A

c. Cluster headache

Cluster headache. Cluster head- ache refers to the tendency of these headaches to cluster in time. They may be distinctly sea- sonal, but the triggering event is unknown. The pain of cluster headache is usually
described as originating in the eye and spread- ing over the temporal area as the headache evolves. In contrast to migraine, men are more often affected than women, and extreme irrita- bility may accompany the headache. The pain usually abates in less than 1 h. Affected persons routinely have autonomic phenomena associ- ated with the headache that include unilateral nasal congestion, tearing from one eye, con- junctival injection, and pupillary constriction. The autonomic phenomena are on the same side of the face as the pain

84
Q

An 81-year-old man with chronic lympho- cytic leukemia develops pain and burning over the right side of his face. Within a few days, a vesiculopapular rash in the distribution of the first division of the tri- geminal nerve appears. The vesicles become encrusted, and the burning associ- ated with the rash abates. Within 1 month the rash has largely resolved, but the man is left with a dull ache over the area of the rash that is periodically punctuated by shooting pains.

Headache:
a. Analgesic overuse headache
b. Atypical facial pain
c. Cluster headache
d. Hypnic headache
e. Migraine with aura
f. Migraine without aura
g. Opthalmoplegic migraine
h. Paroxysmal hemicranias
i. Postherpetic neuralgia
j. Retinal migraine k. Sinusitis
l. Short-lasting unilateral neuralgiform headache with conjunctival injection and tearing (SUNCT)
m. Tension headache
n. Trigeminal neuralgia
o. Vertebrobasilar migraine

A

i. Postherpetic neuralgia

Postherpetic neuralgia. HZV/VZV may reactivate in immunocompromised patients or the severely ill elderly and result in this neuropathic pain syndrome once the acute infection has settled.

85
Q

An obese 37-year-old woman has had a daily headache, worse in the morning, for 1 year. She has episodes of transient visual obscurations affecting each eye and also hears a pulsatile tinnitus. Examination is notable for bilateral papilledema. MRI is normal.

Headache:
a. Analgesic rebound headache
b. Aseptic meningitis
c. Carotid artery dissection
d. Glioblastoma multiforme
e. Idiopathic intracranial hypertension
f. Paroxysmal hemicranias
g. Post-traumatic headache
h. Raeder syndrome
i. Spontaneous intracranial hypotension
j. Thunderclap headache

A

e. Idiopathic intracranial hypertension

ymptoms include headaches, transient visual obscura- tions, progressive visual loss, pulsatile tinni- tus, diplopia, and shoulder and arm pain in obese/overweight females. Papilledema or optic atrophy and occasionally sixth nerve palsies may be present. Diagnoses requires exclusion of structural (e.g. mass lesion or venous sinus obstruction) or other secondary causes, and opening pressure >25 cm H2O on lumbar puncture. Treatment options include serial lumbar puncture, diuretics, ventriculoperitoneal shunting, and optic nerve sheath fenestration.

86
Q

A 42-year-old man presents with a sudden and severe headache associated with nausea during coitus. The headache reaches maxi- mal intensity within 5 s. He has no prior history of headache. Examination is unre- markable. CT head is normal and CSF shows a traumatic tap and cannot exclude subarachnoid hemorrhage. A good quality CT angiogram is negative for aneurysm, dissection or AVM.

Headache:
a. Analgesic rebound headache
b. Aseptic meningitis
c. Carotid artery dissection
d. Glioblastoma multiforme
e. Idiopathic intracranial hypertension
f. Paroxysmal hemicranias
g. Post-traumatic headache
h. Raeder syndrome
i. Spontaneous intracranial hypotension
j. Thunderclap headache

A

j. Thunderclap headache

87
Q

A 29-year-old man relates that he has had recent headaches only when standing up.The headaches resolve quickly when he lies down and are accompanied by mild nausea. His examination is normal

Headache:
a. Analgesic rebound headache
b. Aseptic meningitis
c. Carotid artery dissection
d. Glioblastoma multiforme
e. Idiopathic intracranial hypertension
f. Paroxysmal hemicranias
g. Post-traumatic headache
h. Raeder syndrome
i. Spontaneous intracranial hypotension
j. Thunderclap headache

A

i. Spontaneous intracranial hypotension

Causes of intracranial hypoten- sion include persistent CSF leak after lumbar
puncture, head trauma, neurosurgery, or even pneumonectomy (thoracoarachnoid fis- tula), pituitary tumors, dural tear in the spinal root sleeves, traumatic nerve root avulsion or dehydration.

88
Q

A 45-year-old woman with chronic atrial fibrillation discontinues warfarin treatment and abruptly develops problems with lan- guage comprehension. She is able to pro- duce some intelligible phrases and produces sound quite fluently; however, she is unable to follow simple instructions or to repeat simple phrases. On attempting to write, she becomes very frustrated and agitated. Emergency MRI reveals a lesion of the left temporal lobe that extends into the superior temporal gyrus.

Disorders of language:
a. Akinetic mutism
b. Anomic aphasia
c. Broca’s aphasia
d. Cerebellar mutism
e. Conduction aphasia
f. Global aphasia
g. Mixed transcortical aphasia
h. Transcortical motor aphasia
i. Transcortical sensory aphasia (fluent)
j. Wernicke’s aphasia

A

j. Wernicke’s aphasia

89
Q

A 62-year-old man has had a left hemi- sphere stroke. He has impaired naming and repetition. His speech is nonfluent. Comprehension is preserved.

Disorders of language:
a. Akinetic mutism
b. Anomic aphasia
c. Broca’s aphasia
d. Cerebellar mutism
e. Conduction aphasia
f. Global aphasia
g. Mixed transcortical aphasia
h. Transcortical motor aphasia
i. Transcortical sensory aphasia (fluent)
j. Wernicke’s aphasia

A

c. Broca’s aphasia

90
Q

A 28-year-old woman is hit in the left neck while playing lacrosse. Approximately 2 h later she begins having language difficulties. Her speech is fluent and nonsensical. She cannot understand commands, but repeats well.

Disorders of language:
a. Akinetic mutism
b. Anomic aphasia
c. Broca’s aphasia
d. Cerebellar mutism
e. Conduction aphasia
f. Global aphasia
g. Mixed transcortical aphasia
h. Transcortical motor aphasia
i. Transcortical sensory aphasia (fluent)
j. Wernicke’s aphasia

A

i. Transcortical sensory aphasia (fluent)